SlideShare une entreprise Scribd logo
1  sur  37
19 Phương pháp chứng minh Bất đẳng thức_ Nguyễn Đức Hàn www.VNMATH.com



                                   PHẦN 1
                           CÁC KIẾN THỨC CẦN LƯU Ý



                          A ≥ B ⇔ A − B ≥ 0
     1/Định nghĩa         
                          A ≤ B ⇔ A − B ≤ 0
     2/Tính chất
     + A>B ⇔ B < A
     + A>B và B >C ⇔ A > C
     + A>B ⇒ A+C >B + C
     + A>B và C > D ⇒ A+C > B + D
     + A>B và C > 0 ⇒ A.C > B.C
     + A>B và C < 0 ⇒ A.C < B.C
     + 0 < A < B và 0 < C <D ⇒ 0 < A.C < B.D
     + A > B > 0 ⇒ A n > B n ∀n
     + A > B ⇒ A n > B n với n lẻ
     + A > B ⇒ A n > B n với n chẵn
     + m > n > 0 và A > 1 ⇒ A m > A n
     + m > n > 0 và 0 <A < 1 ⇒ A m < A n
                                 1 1
      +A < B và A.B > 0     ⇒     >
                                 A B

      3/Một số hằng bất đẳng thức

     + A 2 ≥ 0 với ∀ A ( dấu = xảy ra khi A = 0 )
     + An ≥ 0 với ∀ A ( dấu = xảy ra khi A = 0 )
     + A ≥ 0 với ∀A (dấu = xảy ra khi A = 0 )
     + -A <A= A
     + A + B ≥ A + B ( dấu = xảy ra khi A.B > 0)
     + A − B ≤ A − B ( dấu = xảy ra khi A.B < 0)




Sưu tầm và tuyển chọn                          1
19 Phương pháp chứng minh Bất đẳng thức_ Nguyễn Đức Hàn www.VNMATH.com

                               PHẦN II
              CÁC PHƯƠNG PHÁP CHỨNG MINH BẤT ĐẲNG THỨC


      Phương pháp 1 : Dùng định nghĩa
      Kiến thức : Để chứng minh A > B. Ta lập hiệu A –B > 0
                 Lưu ý dùng hằng bất đẳng thức M 2 ≥ 0 với∀ M

      Ví dụ 1 ∀ x, y, z chứng minh rằng :
         a) x 2 + y 2 + z 2 ≥ xy+ yz + zx
         b) x 2 + y 2 + z 2 ≥ 2xy – 2xz + 2yz
       c) x 2 + y 2 + z 2 +3 ≥ 2 (x + y + z)
      Giải:
                                                               1
      a) Ta xét hiệu : x 2 + y 2 + z 2 - xy – yz – zx = .2 .( x 2 + y 2 + z 2 - xy – yz – zx)
                                                               2
      =
       1
       2
          [                              ]
          ( x − y ) 2 + ( x −z ) 2 + ( y − z ) 2 ≥ 0 đúng với mọi x;y;z ∈ R

     Vì (x-y)2 ≥ 0 với∀x ; y Dấu bằng xảy ra khi x=y
         (x-z)2 ≥ 0 với∀x ; z Dấu bằng xảy ra khi x=z
         (y-z)2 ≥ 0 với∀ z; y Dấu bằng xảy ra khi z=y
         Vậy x 2 + y 2 + z 2 ≥ xy+ yz + zx. Dấu bằng xảy ra khi x = y =z
      b)Ta xét hiệu: x 2 + y 2 + z 2 - ( 2xy – 2xz +2yz ) = x 2 + y 2 + z 2 - 2xy +2xz –2yz
      = ( x – y + z) 2 ≥ 0 đúng với mọi x;y;z ∈ R
      Vậy x 2 + y 2 + z 2 ≥ 2xy – 2xz + 2yz đúng với mọi x;y;z ∈ R
      Dấu bằng xảy ra khi x+y=z
      c) Ta xét hiệu: x 2 + y 2 + z 2 +3 – 2( x+ y +z ) = x 2 - 2x + 1 + y 2 -2y +1 + z 2 -2z +1
      = (x-1) 2 + (y-1) 2 +(z-1) 2 ≥ 0. Dấu(=)xảy ra khi x=y=z=1
      Ví dụ 2: chứng minh rằng :
                              2                                           2
         a2 + b2  a + b                    a2 + b2 + c2  a + b + c 
      a)        ≥        ;        b)                   ≥                  c) Hãy tổng quát bài toán
            2     2                             3            3     
      Giải:
                                                  2
                             a2 + b2  a + b 
     a) Ta xét hiệu                   −     
                                2       2 
        2( a 2 + b 2 ) a 2 + 2ab + b 2
                                        = ( 2a 2 + 2b 2 − a 2 − b 2 − 2ab ) = ( a − b ) ≥ 0
                                         1                                   1         2
      =               −
               4              4          4                                   4
                                2
           a +b
              2     2
                        a+b
     Vậy              ≥        .     Dấu bằng xảy ra khi a=b
                2        2 
      b)Ta xét hiệu




Sưu tầm và tuyển chọn                                 2
19 Phương pháp chứng minh Bất đẳng thức_ Nguyễn Đức Hàn www.VNMATH.com

                                                                                                                     [                         ]
                                                                                                            2
                                                 a2 + b2 + c2  a + b + c  1
                                                                           = ( a − b ) + ( b − c ) + ( c − a ) ≥ 0 .Vậy
                                                                                       2           2           2
                                                             −
                                                      3            3      9
                                     2
a2 + b2 + c2  a + b + c 
            ≥           
     3            3     
      Dấu bằng xảy ra khi a = b =c

      c)Tổng quát
                                                                                    2
         a12 + a 2 + .... + a n  a1 + a 2 + .... + a n 
                 2            2
                                ≥                      
                   n                      n            
      Tóm lại các bước để chứng minh A ≥ B theo định nghĩa
      Bước 1: Ta xét hiệu H = A - B
      Bước 2:Biến đổi H=(C+D) 2 hoặc H=(C+D) 2 +….+(E+F) 2
      Bước 3:Kết luận A ≥ B

      Ví dụ 1: Chứng minh ∀m,n,p,q ta đều có : m 2 + n 2 + p 2 + q 2 +1≥ m(n+p+q+1)
      Giải:
        m2              m2              m2              m2       
      ⇔
        4  − mn + n 2  + 
                         4  − mp + p 2  + 
                                           4  − mq + q 2  + 
                                                             4 − m + 1 ≥ 0
                                                                        
                                                                 
                       2                     2                          2                   2
        m     m       m        m 
      ⇔  − n  +  − p  +  − q  +  − 1 ≥ 0 (luôn đúng)
        2     2       2        2     
                          m
                           2 −n =0        m
                          m           n = 2
                           − p=0          m
                          2           p =       m=2
      Dấu bằng xảy ra khi  m        ⇔      2 ⇔
                           −q =0          m    n = p = q = 1
                          2            q=
                          m           m =2
                          2  −1 = 0        2
                          

      Ví dụ 2: Chứng minh rằng với mọi a, b, c ta luôn có : a 4 + b 4 + c 4 ≥ abc(a + b + c)
      Giải: Ta có : a 4 + b 4 + c 4 ≥ abc(a + b + c) , ∀ , b, c > 0
                                                        a
      ⇔ a 4 + b 4 + c 4 − a 2 bc − b 2 ac − c 2 ab ≥ 0
      ⇔ 2a 4 + 2b 4 + 2c 4 − 2a 2 bc − 2b 2 ac − 2c 2 ab ≥ 0
         (
      ⇔ a2 −b2     )   2
                                         (
                           + 2a 2 b 2 + b 2 − c 2     )   2
                                                                               (
                                                              + 2b 2 c 2 + c 2 − a 2            )   2
                                                                                                        + 2a 2 c 2
                                                  − 2a 2 bc − 2b 2 ac − 2c 2 ab ≥ 0
         (
      ⇔ a2 −b2     ) + (b
                       2       2
                                   −c2   ) + (c
                                         2        2
                                                      −a2      )   2
                                                                       + (a 2 b 2 + b 2 c 2 − 2b 2 ac ) + (b 2 c 2 + c 2 a 2 − 2c 2 ab)
                                                                                                        + ( a 2 b 2 + c 2 a 2 − 2a 2 ab) ≥ 0
         (
      ⇔ a2 −b2     ) + (b
                       2       2
                                   −c2   ) + (c
                                         2        2
                                                      −a2      ) + ( ab − bc )
                                                                   2                   2
                                                                                           + ( bc − ac ) + ( ab − ac ) ≥ 0
                                                                                                                2               2


      Đúng với mọi a, b, c.


      Phương pháp 2 : Dùng phép biến đổi tương đương

Sưu tầm và tuyển chọn                                                              3
19 Phương pháp chứng minh Bất đẳng thức_ Nguyễn Đức Hàn www.VNMATH.com
      Kiến thức:
      Ta biến đổi bất đẳng thức cần chứng minh tương đương với bất đẳng thức đúng
hoặc bất đẳng thức đã được chứng minh là đúng.
      Nếu A < B ⇔ C < D , với C < D là một bất đẳng thức hiển nhiên, hoặc đã biết là đúng thì có
bất đẳng thức A < B .

       Chú ý các hằng đẳng thức sau:
                  ( A + B ) 2 = A 2 + 2 AB + B 2
                    ( A + B + C ) 2 = A 2 + B 2 + C 2 + 2 AB + 2 AC + 2 BC
                    ( A + B ) 3 = A 3 + 3 A 2 B + 3 AB 2 + B 3
       Ví dụ 1: Cho a, b, c, d,e là các số thực chứng minh rằng
                      b2
             a) a 2 +     ≥ ab
                       4
             b) a 2 + b 2 + 1 ≥ ab + a + b
             c) a 2 + b 2 + c 2 + d 2 + e 2 ≥ a( b + c + d + e )
       Giải:
             b2
                 ≥ ab ⇔ 4a 2 + b 2 ≥ 4ab ⇔ 4a 2 − 4a + b 2 ≥ 0 ⇔ ( 2a − b ) ≥ 0
                                                                           2
     a) a 2 +
              4
                                                b2
      (BĐT này luôn đúng). Vậy a 2 + ≥ ab (dấu bằng xảy ra khi 2a=b)
                                                 4
      b) a + b + 1 ≥ ab + a + b ⇔ 2(a + b 2 + 1 ) > 2(ab + a + b)
             2     2                           2


      ⇔ a 2 − 2ab + b 2 + a 2 − 2a + 1 + b 2 − 2b + 1 ≥ 0
      ⇔ (a − b) 2 + (a − 1) 2 + (b − 1) 2 ≥ 0      Bất đẳng thức cuối đúng.
       Vậy a 2 + b 2 + 1 ≥ ab + a + b . Dấu bằng xảy ra khi a=b=1
       c) a 2 + b 2 + c 2 + d 2 + e 2 ≥ a( b + c + d + e ) ⇔ 4( a 2 + b 2 + c 2 + d 2 + e 2 ) ≥ 4a( b + c + d + e )
        ⇔ ( a 2 − 4ab + 4b 2 ) + ( a 2 − 4ac + 4c 2 ) + ( a 2 − 4ad + 4d 2 ) + ( a 2 − 4ac + 4c 2 ) ≥ 0
        ⇔ ( a − 2b ) 2 + ( a − 2c ) 2 + ( a − 2d ) 2 + ( a − 2c ) 2 ≥ 0
        Bất đẳng thức đúng vậy ta có điều phải chứng minh
       Ví dụ 2: Chứng minh rằng: ( a10 + b10 )( a 2 + b 2 ) ≥ ( a 8 + b 8 )( a 4 + b 4 )
       Giải:
       (a + b10 )( a 2 + b 2 ) ≥ ( a 8 + b 8 )( a 4 + b 4 ) ⇔ a 12 + a 10 b 2 + a 2 b10 + b12 ≥ a 12 + a 8 b 4 + a 4 b 8 + b12
            10


       ⇔ a 8 b 2 ( a 2 − b 2 ) + a 2 b 8 ( b 2 − a 2 ) ≥ 0 ⇔ a2b2(a2-b2)(a6-b6) ≥ 0
       ⇔ a2b2(a2-b2)2(a4+ a2b2+b4) ≥ 0
       Bất đẳng thứccuối đúng vậy ta có điều phải chứng minh
                                                                         x2 + y2
       Ví dụ 3: cho x.y =1 và x 〉 y                 Chứng minh                   ≥2 2
                                                                          x− y
              x2 + y2
       Giải:          ≥ 2 2 vì :x 〉 y nên x- y 〉 0 ⇒ x2+y2 ≥ 2 2 ( x-y)
               x− y
       ⇒ x +y2- 2 2 x+ 2 2 y ≥ 0 ⇔ x2+y2+2- 2 2 x+ 2 2 y -2 ≥ 0
            2

       ⇔ x2+y2+( 2 )2- 2 2 x+ 2 2 y -2xy ≥ 0 vì x.y=1 nên 2.x.y=2
       ⇒ (x-y- 2 )2 ≥ 0 Điều này luôn luôn đúng . Vậy ta có điều phải chứng minh
       Ví dụ 4: Chứng minh rằng:
       a/ P(x,y)= 9 x 2 y 2 + y 2 − 6 xy − 2 y + 1 ≥ 0 ∀x, y ∈ R

Sưu tầm và tuyển chọn                                        4
19 Phương pháp chứng minh Bất đẳng thức_ Nguyễn Đức Hàn www.VNMATH.com
      b/ a 2 + b 2 + c 2 ≤ a + b + c (gợi ý :bình phương 2 vế)
      c/ Cho ba số thực khác không x, y, z thỏa mãn:
                         x. y.z = 1
                      1 1 1
                       + + < x+ y+z
                      x y z
                      

      Chứng minh rằng :có đúng một trong ba số x,y,z lớn hơn 1
       Giải: Xét (x-1)(y-1)(z-1)=xyz+(xy+yz+zx)+x+y+z-1
                                 1    1   1                        1   1   1          1   1   1
      =(xyz-1)+(x+y+z)-xyz( x + y + z )=x+y+z - ( + + ) > 0 (vì x + y + z < x+y+z
                                                      x y z
theo gt)
     ⇒ 2 trong 3 số x-1 , y-1 , z-1 âm hoặc cả ba sỗ-1 , y-1, z-1 là dương.
      Nếu trường hợp sau xảy ra thì x, y, z >1 ⇒ x.y.z>1 Mâu thuẫn gt x.y.z=1 bắt
buộc phải xảy ra trường hợp trên tức là có đúng 1 trong ba số x ,y ,z là số lớn hơn 1
                                              a        b       c
      Ví dụ 5: Chứng minh rằng : 1 < a + b + b + c + a + c < 2
         Giải:
                                      1        1          a        a
         Ta có :   a+b< a+b+c⇒             >         ⇒       >          (1)
                                     a+b a+b+c          a+b a+b+c
                               b         b             c         c
         Tương tự ta có    : b + c > a + b + c (2) , a + c > a + b + c (3)
         Cộng vế theo vế các bất đẳng thức (1), (2), (3), ta được :
          a      b       c
             +       +      > 1 (*)
         a+b b+c a+c
                               a       a+c
         Ta có : a < a + b ⇒ a + b < a + b + c (4)
                        b       a+b                        c   c+b
         Tương tự : b + c < a + b + c (5) ,                  <
                                                          c+a a+b+c
                                                                               ( 6)

         Cộng vế theo vế các bất đẳng thức (4), (5), (6), ta được :
             a   b   c
               +   +    <2         (**)
            a+b b+c a+c
                                          a       b        c
      Từ (*) và (**) , ta được : 1 < a + b + b + c + a + c < 2 (đpcm)


      Phương pháp 3:         Dùng bất đẳng thức phụ
      Kiến thức:
      a) x 2 + y 2 ≥ 2 xy
      b) x + y ≥ xy dấu( = ) khi x = y = 0
            2      2


      c) ( x + y ) 2 ≥ 4 xy
        a    b
      d) + ≥ 2
        b    a
      Ví dụ 1 Cho a, b ,c là các số không âm chứng minh rằng
                    (a+b)(b+c)(c+a) ≥ 8abc
      Giải: Dùng bất đẳng thức phụ: ( x + y ) 2 ≥ 4 xy

Sưu tầm và tuyển chọn                                 5
19 Phương pháp chứng minh Bất đẳng thức_ Nguyễn Đức Hàn www.VNMATH.com
     Tacó        ( a + b ) 2 ≥ 4ab ; ( b + c ) 2 ≥ 4bc ; ( c + a ) 2 ≥ 4ac
      ⇒ ( a + b ) 2 ( b + c ) 2 ( c + a ) 2 ≥ 64a 2 b 2 c 2 = ( 8abc ) 2 ⇒ (a+b)(b+c)(c+a) ≥ 8abc
        Dấu “=” xảy ra khi a = b = c
     Phương pháp 4:            Bất đẳng thức Cô sy
     Kiến thức:
     a/ Với hai số không âm : a, b ≥ 0 , ta có: a + b ≥ 2                           ab .   Dấu “=” xảy ra khi a=b
     b/ Bất đẳng thức mở rộng cho n số không âm :
      a1 + a 2 + ... + a n ≥ n n a1 a 2 ..a n
                                                  n
                       a + a 2 + ... + a n 
     ⇔ a1 a 2 ..a n ≤  1                   
                               n           
     Dấu “=” xảy ra khi a1 = a 2 = ... = a n
     Chú ý : ta dùng bất đẳng thức Côsi khi đề cho biến số không âm.
                                                           2x   4x     2x     3
     Ví dụ 1 : Giải phương trình :                            + x  + x      =
                                                          4 +1 2 +1 2 + 4
                                                           x              x
                                                                              2

                                                                                     a = 2x
     Giải : Nếu đặt t =2x               thì pt trở thành pt bậc 6 theo t nên ta đặt            , a, b > 0
                                                                                      b = 4 x

                                                            a     b        1       3
     Khi đó phương trình có dạng : b + 1 + a + 1 + a + b = 2
     Vế trái của phương trình:
                                      a         b         1       
                                    =      + 1 +     + 1 +     + 1 − 3
                                      b +1   a +1   a + b 
                                      a + b +1  a + b +1  a + b +1
                                    =          +         +         −3
                                      b +1   a +1   a + b 
                                                     1          1          1 
                                   = ( a + b + c)           +          +     −3
                                                     b +1 a +1 a + b 

                                   [ ( b + 1) + ( a + 1) + ( a + b ) ] 1 + 1 + 1  − 3
                                                                                       
                                                                       b +1 a +1 a + b 

                                   1 3                                          3                   3
                              ≥      3 ( a + 1)( b + 1)( a + b ) .                             −3 =
                                   2                               3 ( a + 1)( b + 1)( a + b )      2
     Vậy phương trình tương đương với :
      a + 1 = b + 1 = a + b ⇔ a = b = 1 ⇔ 2x = 4x = 1 ⇔ x = 0 .
                                                                                                        x    y   z
     Ví dụ 2 : Cho x, y , z > 0 và x + y + z = 1. Tìm GTLN của P = x + 1 + y + 1 + z + 1
                               1          1           1
     Giải : P = 3- ( x + 1 + y + 1 + z + 1 ) = 3 – Q. Theo BDT Côsi , nếu a, b, c > 0 thì




Sưu tầm và tuyển chọn                                            6
19 Phương pháp chứng minh Bất đẳng thức_ Nguyễn Đức Hàn www.VNMATH.com
                a + b + c ≥ 3 3 abc
             1 1 1              1
              + + ≥ 33
             a b c             abc
                            1 1 1
             ⇒ ( a + b + c ) + +  ≥ 9
                            a b c
               1 1 1              9
             ⇒ + + ≥
               a b c a +b+c
                   1         1      1   9                        9                              9        3
     Suy ra Q = x + 1 + y + 1 + z + 1 ≥ 4 ⇒             -Q ≤ − 4 nên P = 3 – Q ≤ 3- 4 = 4
                   3                      1
     Vậy max P = 4 .khi x = y = z = 3 .
                                                                       1       1      1     a+b+c
     Ví dụ 3:      Cho a, b, c >0 . Chứng minh rằng:                       + 2    + 2     ≤
                                                                     a + bc b + ac c + ab
                                                                      2
                                                                                             2abc
     Giải: Áp dụng bất đẳng thức Côsi ta có :
                                   2      1  1 1   1 
         a 2 + +bc ≥ 2a bc ⇒           ≤    ≤  + 
                                a + +bc a bc 2  ab ac 
                                  2


     Tương tự :
                      2       1    1 1   1 
                          ≤       ≤  + 
                  b + + ac b ac 2  bc ab 
                    2


                      2       1    1 1   1
                          ≤       ≤  + 
                  c + + ab c ab 2  ac bc 
                   2


                        2       2        2      a+b+c
                  ⇒ 2       + 2      + 2      ≤
                     a + bc b + + ac c + + ab    2abc
     Dấu “=” xảy ra khi a = b = c.
                                                           a              b     c
     Ví dụ 4 : CMR trong tam giác ABC : b + c − a + c + a − b + a + b − c ≥ 3 (*)
     Giải : Theo bất đẳng thức Côsi :
             a         b       c                        abc
                  +        +        ≥ 33                                     (1)
         b + c − a c + a −b a +b −c      (b + c − a )(c + a − b)(a + b − c )
     Cũng theo bất đẳng thức Côsi :
                                      1
          (b + c − a )(c + a − b) ≤     (b + c − a + c + a − b) = c (2)
                                      2
     Viết tiếp hai BDT tương tự (2) rồi nhân với nhau sẽ được
         (b + c − a )(c + a − b)(a + b − c) ≤ abc
                            abc
         →                                      ≥ 1 (3)
            (b + c − a )(c + a − b)(a + b − c )
     Từ (1),(3) suy ra (*). Dấu “=” xảy ra khi a = b = c hay ABC là đều .
     Ví dụ 5:
         0< a≤ b≤ c                                               ( a + c)         2

     Cho               . Chứng minh rằng: ( + by + cz )  + +  ≤
                                                           x y z
                                                                                      ( x + y + z) 2
          0 < x, y , z
                                                         a b c       4ac


     Giải: Đặt f ( x) = x 2 − (a + c) x + ac = 0 có 2 nghiệm a,c
     Mà: a ≤ b ≤ c ⇒ f (b) ≤ 0 ⇔ b 2 − (a + c)b + ac ≤ 0




Sưu tầm và tuyển chọn                                  7
19 Phương pháp chứng minh Bất đẳng thức_ Nguyễn Đức Hàn www.VNMATH.com
             ac                   y
      ⇔b+       ≤ a + c ⇔ yb + ac ≤ ( a + c ) y
             b                    b
                 x           y             z
      ⇒  xa + ac  + ( yb + ac ) + ( zc + ac ) ≤ ( a + c ) x + ( a + c ) y + ( a + c) z
                 a           b             c
                          x y z
      ⇒ xa + yb + zc + ac + +  ≤ ( a + c )( x + y + z )
                          a b c
     Theo bất đẳng thức Cauchy ta có:
      ⇒2     ( xa + yb + zc ) ac x + y + z  ≤ ( a + c )( x + y + z )
                                           
                                  a     b     c
                            x y z
      ⇔ 4( xa + yb + zc ) ac + +  ≤ ( a + c ) ( x + y + z )
                                               2              2

                            a b c
                            x y z  ( a + c)
                                                            2
      ⇔ ( xa + yb + zc ) ac + +  ≤                            ( x + y + z ) 2 (đpcm)
                           a b c      4ac


     Phương pháp 5                              Bất đẳng thức Bunhiacopski
     Kiến thức:
     Cho 2n số thực ( n ≥ 2 ):                a1 , a 2 ,...a n , b1 , b2 ,..., bn .      Ta luôn có:
      (a1b1 + a 2 b2 + ... + a n bn ) ≤ (a12 + a 2 + ... + a n )(b12 + b2 + ... + bn )
                                     2           2           2          2          2


                                          a         a             a
     Dấu “=” xảy ra khi ⇔ b = b = .... = b
                           1   2           n

                           1   2           n

             b1     b2             bn
     Hay a = a = .... = a (Quy ước : nếu mẫu = 0 thì tử = 0 )
          1   2          n

     Chứng minh:
             a = a 2 + a 2 + ... + a 2
                  1     2           n
     Đặt 
             b = b1 + b2 + ... + bn
                    2   2          2
            
          • Nếu a = 0 hay b = 0: Bất đẳng thức luôn đúng.
          • Nếu a,b > 0:
                    ai        b
     Đặt: α i =        , β i = i ( i = 1,2,...n ) ,     Thế thì: α 12 + α 22 + ... + α n2 = β 12 + β 22 + ... + β n2
                    a          b
     Mặt khác: α i β i        ≤
                                  2
                                    (α i + β i2 )
                                  1 2

                                                  1 2                         1
                  α1 β1 + α 2 β 2 + ... + α n β n ≤ (α1 + α 2 + .... + α n ) + ( β12 + β 22 + ... + β n2 ) ≤ 1
                                                            2            2

     Suy ra:                                      2                           2
                  ⇒ a1b1 + a 2 b2 + ... + a n bn ≤ a.b
     Lại có:      a1b1 + a 2 b2 +... + a n bn ≤ a1b1 + a 2 b2 +... + a n bn

     Suy ra:      (a1b1 + a 2 b2 + ... + a n bn ) 2 ≤ (a12 + a 2 + ... + a n )(b12 + b2 + ... + bn )
                                                               2           2          2          2


                                    α i = β i ( ∀i = 1,2,..., n )  a   a          a
     Dấu”=” xảy ra ⇔                                              ⇔ 1 = 2 = .... = n
                                   α 1 β 1 ....α n β n cùng dáu    b1 b2          bn
     Ví dụ 1 :
                                                                                       1
     Chứng minh rằng: ∀x ∈R , ta có:                             sin 8 x + cos 8 x ≥
                                                                                       8
     Giải: Ta có: sin 2 x + cos 2 x = 1, ∀x ∈ R
     Theo bất đẳng thức Bunhiacopski, ta có:


Sưu tầm và tuyển chọn                                                 8
19 Phương pháp chứng minh Bất đẳng thức_ Nguyễn Đức Hàn www.VNMATH.com
         1 = (sin 2 x.1 + cos 2 x.1) ≤ (sin 4 x + cos 4 x )(12 + 12 )
          1
         ⇔  ≤ sin 4 x + cos 4 x
          2
         ⇒ ≤ (sin 4 x + cos 4 x )
          1                       2

          4
         Sử dụng bất đẳng thức Bunhiacopski một lần nữa:
              ≤ (sin 4 x.1 + cos 4 x.1)
           1                            2
         ⇔
           4
         ⇔ ≤ (sin 8 x + cos 8 x )(12 + 12 )
           1
           4
         ⇔ (sin 4 x + cos 4 x ) ≥
                                  1
                                  8
         Ví dụ 2: Cho tam giác ABC có các góc A,B,C nhọn. Tìm GTLN của:
         P = 1 + tan A. tan B + 1 + tan B. tan C + 1 + tan C. tan A
         Giải:
         * Bất đẳng thức Bunhiacopski mở rộng
         Cho m bộ số, mỗi bộ số gồm n số không âm:                          ( a i , bi ,..., ci )(i =1,2,...., m)
         Thế thì:
(a1 a 2 ...a m + b1b2 ...bm + ... + c1c 2 ...c m ) 2 ≤ (a1m + b1m + ... + c1m )(a 2 + b2 + ... + c 2 )(a m + bm + ... + c m )
                                                                                  m    m            m     m      m        m


           Dấu”=” xảy ra ⇔ ∃ bô số (a,b,….,c) sao cho: với mỗi i = 1,2,…,m thì ∃ t i sao
cho: a = t i ai , b = t i bi ,..., c = t i ci , Hay a1 : b1 : ... : c1 = a 2 : b2 : ... : c 2 = a n : bn : ...cn

                                 a12 + a 2 + ... + a n = 3
                                          2           2

         Ví dụ 1: Cho            
                                      n ∈ Z,n ≥ 2
                                          a1 a 2          a
         Chứng minh rằng:                   +    + .... + n < 2
                                          2   3          n +1
         Giải:
                                    1        1                   1
                                       <              =
         ∀ ∈N
          k         *
                        ta có:      k2            1           1    1
                                           k2 −            k −  k + 
                                                  4           2    2
             1          1         1
         ⇒      <            −
             k2        1              1
                    k−           k+
                       2              2
                                                                       
             1   1         1  1    1   1     1          1       1 
         ⇒ 2 + 2 + ... + 2 <                           
                              3 − 5  +  5 − 7  + ... +       −     
                                                            1     1
                                                                    n+ 
            2    3         n
                              2     2  2      2         n−         
                                                               2     2
            1      1     2
         =     −       <
           3         1 3
             2 n+
                     2
         Do đó theo bất đẳng thức Bunhiacopski:
          a1 a 2          a                                     1     1         1     2
            +    + .... + n ≤ a12 + a 2 + ... + a n
                                      2           2
                                                                    + 2 + ... + 2 < 3   < 2 (đpcm)
          2   3          n +1                                   2 2
                                                                     3         n      3
         Ví dụ 2:           Cho 4 số a,b,c,d bất kỳ chứng minh rằng:
                                 ( a + c) 2 + (b + d ) 2 ≤ a 2 + b 2 + c 2 + d 2
         Giải: Dùng bất đẳng thức Bunhiacopski: Tacó                              ac+bd ≤ a 2 + b 2 . c 2 + d 2


Sưu tầm và tuyển chọn                                               9
19 Phương pháp chứng minh Bất đẳng thức_ Nguyễn Đức Hàn www.VNMATH.com
mà ( a + c ) 2 + ( b + d ) 2 = a 2 + b 2 + 2( ac + bd ) + c 2 + d 2 ≤ ( a 2 + b 2 ) + 2 a 2 + b 2 . c 2 + d 2 + c 2 + d 2
         ⇒ (a + c) 2 + (b + d ) 2 ≤ a 2 + b 2 + c 2 + d 2
         Ví dụ 3: Chứng minh rằng : a 2 + b 2 + c 2 ≥ ab + bc + ac
         Giải: Dùng bất đẳng thức Bunhiacopski
         Cách 1: Xét cặp số (1,1,1) và (a,b,c) ta có (12 + 12 + 12 )(a 2 + b 2 + c 2 ) ≥ (1.a + 1.b + 1.c ) 2
         ⇒ 3 ( a 2 + b 2 + c 2 ) ≥ a 2 + b 2 + c 2 + 2( ab + bc + ac )
         ⇒ a 2 + b 2 + c 2 ≥ ab + bc + ac Điều phải chứng minh Dấu bằng xảy ra khi a=b=c



         Phương pháp 6:                         Bất đẳng thức Trê- bư-sép
         Kiến thức:
                a1 ≤ a2 ≤ ..... ≤ an           a1 + a 2 + ... + a n b1 + b2 + .... + bn a1b1 + a 2 b2 + .... + a n bn
         a)Nếu                           thì                       .                   ≤                                .
                b1 ≤ b2 ≤ ..... ≤ bn
                                                         n                   n                        n

                                                   a1 = a 2 = .... = a n
         Dấu ‘=’ xảy ra khi và chỉ khi b
                                                    1 = b2 = .... = bn

                a1 ≤ a2 ≤ ..... ≤ an
         b)Nếu                       thì
                b1 ≥ b2 ≥ ..... ≥ bn
         a1 + a 2 + ... + a n b1 + b2 + .... + bn a1b1 + a 2 b2 + .... + a n bn
                             .                   ≥
                  n                   n                        n
                                                   a1 = a 2 = .... = a n
         Dấu ‘=’ xảy ra khi và chỉ khi b
                                                    1 = b2 = .... = bn
         Ví dụ 1: Cho ∆ABC có 3 góc nhọn nội tiếp đường tròn bán kính R = 1 và
         sin A. sin 2a + sin B. sin 2 B + sin C. sin 2C 2 S
                                                       =    .
                     sin A + sin B + sin C               3
         S là diện tích tan giác. chứng minh rằng ∆ABC là tam giác đều.
                                                                                          π
         Giải: Không giảm tính tổng quát ta giả sư                    0< A≤B≤C <            . Suy   ra:
                                                                                          2

          sin A ≤ sin B ≤ sin C
         
          sin 2a ≤ sin 2B ≤ sin 2C
         Áp dụng BĐT trebusep ta được:
         ( sin A + sin B + sin C )( sin 2 A + sin 2 B + sin 2C ) ≥
         ≥ 3( sin A. sin 2 A + sin B. sin 2 B + sin C. sin 2C )
             sin A. sin 2 A + sin B. sin 2 B + sin C. sin 2C 1
         ⇔                                                  ≤ (sin 2 A + sin 2 B + sin 2C )
                          sin A + sin B + sin C              3
                                  sin A = sin B = sin C
         Dấu ‘=’ xảy ra ⇔ sin 2 A = sin 2 B = sin 2C ⇔ ∆ABC dêu
                                  
         Mặt khác:


Sưu tầm và tuyển chọn                                        10
19 Phương pháp chứng minh Bất đẳng thức_ Nguyễn Đức Hàn www.VNMATH.com
      sin 2 A + sin 2 B + sin 2C = 2 sin( A + B ). cos( A − B ) + sin 2C
      = 2 sin C [cos( A − B ) + cos C ] = 2 sin C [cos( A − B ) − cos( A + B )]
      = 2 sin C.2 sin A. sin B = 4 sin A sin B sin C
      = (2 R sin A)(2 R sin B ). sin C = a.b. sin C = 2 S     (2)
     Thay (2) vào (1) ta có
             sin A. sin 2a + sin B. sin 2 B + sin C. sin 2C 2S
                                                           ≤   .
                         sin A + sin B + sin C               3
     Dấu   ‘=’ xảy ra ⇔ ∆ ABC đều.

     Ví dụ 2(HS tự giải):
                                                       1 1 1
     a/          Cho a,b,c>0 và a+b+c=1           CMR:  + + ≥9
                                                       a b c
     b/          Cho x,y,z>0 và x+y+z=1           CMR:x+2y+z ≥ 4(1 − x)(1 − y )(1 − z )
     c/          Cho a>0 , b>0, c>0
                    a   b   c   3
     CMR:             +   +   ≥
                   b+c c+a a+b 2
                                                                                     1
     d)Cho x ≥ 0 ,y ≥ 0 thỏa mãn 2 x − y = 1                     ;CMR:       x+y ≥
                                                                                     5
                                                                                      a3   b3   c3   1
     Ví dụ 3: Cho a>b>c>0 và a 2 + b 2 + c 2 = 1 . Chứng minh rằng                       +    +    ≥
                                                                                     b+c a+c a+b 2
     Giải:
                                           
                                               a2 ≥ b2 ≥ c2
     Do a,b,c đối xứng ,giả sử a ≥ b ≥ c ⇒  a ≥ b ≥ c
                                           b + c a + c a + b
                                           
     Áp dụng BĐT Trê- bư-sép ta có
         a         b         c     a2 + b2 + c2  a    b     c  1 3 1
     a2.    + b2.     + c2.      ≥              .   +    +    = . =
       b+c        a+c       a+b         3        b+c a+c a+b 3 2 2
           a3     b3     c3    1                                 1
     Vậy      +       +      ≥        Dấu bằng xảy ra khi a=b=c=
          b+c a+c a+b 2                                           3
     Ví dụ 4:           Cho a,b,c,d>0 và abcd =1 .Chứng minh rằng :
     a + b + c + d 2 + a( b + c ) + b( c + d ) + d ( c + a ) ≥ 10
       2     2      2


     Giải: Ta có a 2 + b 2 ≥ 2ab
                     c 2 + d 2 ≥ 2cd
                                 1                1 1
     Do abcd =1 nên cd =             (dùng x + ≥ )
                                ab                 x 2
                                                      1
     Ta có a 2 + b 2 + c 2 ≥ 2(ab + cd ) = 2(ab + ) ≥ 4 (1)
                                                     ab
     Mặt khác: a( b + c ) + b( c + d ) + d ( c + a ) = (ab+cd)+(ac+bd)+(bc+ad)
              1             1         1 
      =  ab +  +  ac +  +  bc +  ≥ 2 + 2 + 2
              ab           ac        bc 
     Vậy a + b + c + d + a( b + c ) + b( c + d ) + d ( c + a ) ≥ 10
           2    2     2     2




     Phương pháp7                       Bất đẳng thức Bernouli
     Kiến thức:

Sưu tầm và tuyển chọn                                 11
19 Phương pháp chứng minh Bất đẳng thức_ Nguyễn Đức Hàn www.VNMATH.com
      a)Dạng nguyên thủy: Cho a ≥ -1, 1 ≤                                            n ∈ Z thì (1 + a )   n
                                                                                                              ≥ 1 + na .   Dấu ‘=’ xảy ra khi
                 =0
                 a
và chỉ khi n =1
           
      b) Dạng mở rộng:
      - Cho a > -1, α ≥ 1 thì (1 + a ) α ≥ 1 + na . Dấu bằng xảy ra khi và chỉ khi a = 0.
                                                                                       =0
                                                                                       a
      - cho a ≥ −1,0 < α < 1 thì (1 + a ) α ≤ 1 + na . Dấu bằng xảy ra khi va chỉ khi  =1 .
                                                                                       α                                          
      Ví dụ 1 : Chứng minh rằng                                   a b + b a > 1, ∀a, b > 0 .
      Giải
          - Nếu a ≥ 1 hay b ≥ 1 thì BĐT luôn đúng
          - Nếu 0 < a,b < 1
          Áp dụng BĐT Bernouli:
                                b(1 − a ) a + b
              b           b
          1     1− a 
                = 1+
                          <1+         <
                a                    a                    a             a
                         a
                ⇒ ab >      .
                       a +b
                                                                          b
                Chứng minh tương tự: b a                           >         .      Suy ra a b + b a > 1          (đpcm).
                                                                         a+b
      Ví dụ 2: Cho a,b,c > 0.Chứng minh rằng
                                                  5
      a5 + b5 + c5  a + b + c 
           3
                  ≥
                         3
                                                     .   (1)
                              
      Giải
                                    5                     5                     5

      (1) ⇔  3a  +  3b  +  3c  ≥ 3
                              
            a+b+c a+b+c a+b+c
      Áp dụng BĐT Bernouli:
                                     5( b + c − 2a )
                            5                             5
       3a        b + c − 2a 
              = 1 +
        a +b+c        a +b+c 
                                ≥1+
                                        a +b+c
                                                                                          (2)
                 
      Chứng minh tương tự ta đuợc:

                      5( c + a − 2b )
                            5
       3b      
                ≥1+
        a +b +c         a +b +c
                                                                   (3)
      
                      5( a + b − 2c )
                            5
       3c      
                ≥1+
        a +b +c         a +b +c
                                                                   (4)
      
      Cộng (2) (3) (4) vế theo vế ta có
                        5                     5                      5
       3a   3b   3c 
            +     +     ≥ 3 ⇒ (đpcm)
      a+b+c a+b+c a+b+c
      Chú ý: ta có bài toán tổng quát sau đây:
      “Cho a1 , a 2 ,...a n > 0; r ≥1. Chứng minh rằng
                                                                            r
        a1r + a 2 + .... + a n  a1 + a 2 + .... + a n 
                r            r
                               ≥
                                                       .
                                                       
                  n                      n            
      Dấu ‘=’ ⇔ a1 = a 2 = .... = a n .(chứng minh
                                              tương tự bài trên).
      Ví dụ 3: Cho 0 ≤ x, y, z ≤1 . Chứng minh rằng
      (2   x
               + 2 y + 2 z )( 2 − x + 2 − y + 2 − z ) ≤
                                                              81
                                                              8
                                                                 .
      Giải
      Đặt a = 2 x , b = 2 y , c = 2 z (1 ≤ a, b, c ≤ 2 ) .

Sưu tầm và tuyển chọn                                                               12
19 Phương pháp chứng minh Bất đẳng thức_ Nguyễn Đức Hàn www.VNMATH.com
      1 ≤ a ≤ 2 ⇒ ( a − 1)( a − 2 ) ≤ 0
                                                                   2
                        ⇒ a 2 − 3a + 2 ≤ 0 ⇒ a +                     ≤ 3 (1)
                                                                   a
     Chứng minh tương tự:
        2
      b+  ≤3                ( 2)
        b
        2
      c+ ≤3                 (3)
        c
     Cộng (1) (2) (3) vế theo vế ta được
                            1 1 1  côsi              1 1 1
      9 ≥ ( a + b + c ) + 2 + +  ≥ 2 ( a + b + c ) 2 + + 
                           a b c                    a b c
                81               1 1 1
      ⇒            ≥ (a + b + c) + +  ⇒ (đpcm)
                8               a b c
     Chú ý: Bài toán tổng quát dạng này
     “ Cho n số x1 , x 2 ,...., x n ∈[ a, b] , c > 1
     Ta luôn có:
                                                                                      ) ≤ [ n( c + cb )   ]   2

      (c                                )( c
                                                                                                   a
                                               − x1        − x2                − xn
           x1
                + c + .... + c
                   x2              xn
                                                      +c          + .... + c
                                                                                               4c a + b

     Phương pháp 8:                                    Sử dụng tính chất bắc cầu

     Kiến thức: A>B và B>C thì A>C
     Ví dụ 1: Cho a, b, c ,d >0 thỏa mãn a> c+d , b>c+d
      Chứng minh rằng ab >ad+bc
     Giải:

                   a > c + d    a − c > d > 0
     Tacó                    ⇒                ⇒                                                         (a-c)(b-d) > cd
                   b > c + d    b − d > c > 0
     ⇔            ab-ad-bc+cd >cd ⇔ ab> ad+bc                                                                     (điều phải chứng minh)
                                                   5        1 1 1  1
     Ví dụ 2: Cho a,b,c>0 thỏa mãn a + b + c = . Chứng minh
                                       2   2   2
                                                             + + <
                                                   3        a b c abc
     Giải: Ta có :( a+b- c) = a +b +c +2( ab –ac – bc) 〉 0
                           2   2  2  2

                 1 2 2 2
      ⇒ ac+bc-ab 〈 ( a +b +c )
                 2
                   5                                 1 1 1                                                                        1
      ⇒ ac+bc-ab ≤ 〈 1 Chia hai vế cho abc > 0 ta có  + − 〈
                   6                                 a b c                                                                       abc
     Ví dụ 3: Cho 0 < a,b,c,d <1 Chứng minh rằng (1-a).(1-b) ( 1-c).(1-d) > 1-a-b-c-d
     Giải: Ta có (1-a).(1-b) = 1-a-b+ab
     Do a>0 , b>0 nên ab>0 ⇒ (1-a).(1-b) > 1-a-b        (1)
     Do c <1 nên 1- c >0 ta có  ⇒ (1-a).(1-b) ( 1-c) > 1-a-b-c
     ⇒ (1-a).(1-b) ( 1-c).(1-d) > (1-a-b-c) (1-d) =1-a-b-c-d+ad+bd+cd
     ⇒ (1-a).(1-b) ( 1-c).(1-d) > 1-a-b-c-d (Điều phải chứng minh)

     Ví dụ 4: Cho 0 <a,b,c <1 . Chứng minh rằng: 2a 3 + 2b 3 + 2c 3 < 3 + a 2 b + b 2 c + c 2 a

Sưu tầm và tuyển chọn                                                                      13
19 Phương pháp chứng minh Bất đẳng thức_ Nguyễn Đức Hàn www.VNMATH.com
     Giải:
     Do a < 1 ⇒ a 2 < 1 và
     Ta có (1 − a 2 ).(1 − b ) < 0 ⇒ 1-b- a 2 + a 2 b > 0 ⇒ 1+ a 2 b 2 > a 2 + b
     mà 0< a,b <1 ⇒ a 2 > a 3 , b 2 > b 3
     Từ (1) và (2) ⇒ 1+ a 2 b 2 > a 3 + b 3 . Vậy a 3 + b 3 < 1+ a 2 b 2
     Tương tự b 3 + c 3 ≤ 1 + b 2 c
                       c 3 + a3 ≤ 1 + c2a
     Cộng các bất đẳng thức ta có : 2a 3 + 2b 3 + 2c 3 ≤ 3 + a 2 b + b 2 c + c 2 a
     Ví dụ 5 Chứng minh rằng : Nếu a 2 + b 2 = c 2 + d 2 = 1998 thì ac+bd =1998
     Giải:
     Ta có (ac + bd) 2 + (ad – bc ) 2 = a 2 c 2 + b 2 d 2 + 2abcd + a 2 d 2 + b 2 c 2 - 2abcd =
     = a2(c2+d2)+b2(c2+d2) =(c2+d2).( a2+ b2) = 19982
     rõ ràng (ac+bd)2 ≤ ( ac + bd ) 2 + ( ad − bc ) 2 = 1998 2 ⇒ ac + bd ≤ 1998
     Ví dụ 6 (HS tự giải) :
     a/ Cho các số thực : a1; a2;a3 ….;a2003 thỏa mãn : a1+ a2+a3 + ….+a2003 =1
                                                        1
c hứng minh rằng : a 1 + a 2 + a32 + .... + a 2003 ≥
                     2     2                  2

                                                       2003
       b/ Cho a;b;c ≥ 0 thỏa mãn :a+b+c=1
                       1      1       1
Chứng minh rằng: ( − 1).( − 1).( − 1) ≥ 8
                       a      b       c


       Phương pháp 9:           Dùng tính chất của tỷ số
       Kiến thức
       1) Cho a, b ,c là các số dương thì
               a        a a+c
        a – Nếu  > 1 thì >
               b        b b+c
               a        a a+c
        b – Nếu < 1 thì <
               b        b b+c
       2) Nếu b,d >0 thì từ
              a c a a+c c
               < ⇒ <   <
              b d b b+d d
`
       Ví dụ 1: Cho a,b,c,d > 0 .Chứng minh rằng
               a     b     c       d
        1<        +     +      +       <2
             a+b+c b+c+d c+d +a d +a+b
       Giải: Theo tính chất của tỉ lệ thức ta có
           a         a     a+d
               <1⇒       <                             (1)
        a+b+c      a+b+c a+b+c+d
                    a       a
       Mặt khác :       >                                     (2)
                  a+b+c a+b+c+d
       Từ (1) và (2) ta có

           a        a     a+d
                <      <                               (3)
        a+b+c+d   a+b+c a+b+c+d

Sưu tầm và tuyển chọn                             14
19 Phương pháp chứng minh Bất đẳng thức_ Nguyễn Đức Hàn www.VNMATH.com
     Tương tự ta có
         b         b        b+a
              <         <                            (4)
      a+b+c+d b+c+d a+b+c+d
            c         c       b+c
                 <        <                              (5)
        a+b+c+d c+d +a a+b+c+d
           d         d       d +c
                <         <                              (6)
       a+b+c+d d +a+b a+b+c+d
      cộng vế với vế của (3); (4); (5); (6) ta có
            a        b        c          d
      1<        +         +         +        < 2 điều phải chứng minh
         a+b+c b+c+d c+d +a d +a+b
                   a c                                   a ab + cd c
      Ví dụ 2 :Cho: < và b,d > 0 .Chứng minh rằng < 2             <
                   b d                                   b b +d2 d
                  a c      ab cd       ab ab + cd cd c
      Giải: Từ < ⇒ 2 < 2 ⇒ 2 < 2                  <    =
                  b d      b   d       b    b +d2 d2 d
              a ab + cd c
      Vậy      <        <        điều phải chứng minh
              b b2 + d 2 d

      Ví dụ 3 : Cho a;b;c;dlà các số nguyên dương thỏa mãn : a+b = c+d =1000
                           a   b
tìm giá trị lớn nhất của +
                           c   d
                                                               a   b     a  b        a a+b b
      Giải: Không mất tính tổng quát ta giả sử :                 ≤   Từ : ≤      ⇒    ≤   ≤
                                                               c   d     c  d        c c+d d
a
  ≤ 1 vì a+b = c+d
c
                            b        a b
      a/ Nếu :b ≤ 998 thì     ≤ 998 ⇒ + ≤ 999
                            d        c d
                                a b 1 999
      b/Nếu: b=998 thì a=1 ⇒ + = +         Đạt giá trị lớn nhất khi d= 1; c=999
                                 c d c  d
                               a b       1
      Vậy giá trị lớn nhất của + =999+     khi a=d=1; c=b=999
                               c d     999

      Phương pháp 10: Phương pháp làm trội
      Kiến thức:
      Dùng các tính bất đẳng thức để đưa một vế của bất đẳng thức về dạng tính được
tổng hữu hạn hoặc tích hữu hạn.
      (*) Phương pháp chung để tính tổng hữu hạn : S = u1 + u2 + .... + un
      Ta cố gắng biến đổi số hạng tổng quát u k về hiệu của hai số hạng liên tiếp nhau:
             u k = ak − ak +1
      Khi đó :S = ( a1 − a2 ) + ( a2 − a3 ) + .... + ( an − an+1 ) = a1 − an+1
      (*) Phương pháp chung về tính tích hữu hạn: P = u1u2 ....un
                                                                                  k    a
      Biến đổi các số hạng u k về thương của hai số hạng liên tiếp nhau: u k = a
                                                                                 k +1

                      a a          a    a
      Khi đó P = a . a ..... a = a
                  1   2        n    1

                  2   3       n +1 n +1




Sưu tầm và tuyển chọn                               15
19 Phương pháp chứng minh Bất đẳng thức_ Nguyễn Đức Hàn www.VNMATH.com
     Ví dụ 1: Với mọi số tự nhiên n >1 chứng minh rằng
           1     1      1               1       3
             <      +        + .... +        <
           2 n +1 n + 2               n+n 4
                        1          1       1
     Giải: Ta có             >         =             với k = 1,2,3,…,n-1
                      n + k n + n 2n
               1     1            1      1            1   n 1
     Do đó:       +      + ... +      >      + ... +    =   =
             n +1 n + 2          2n 2n               2 n 2n 2

     Ví dụ 2: Chứng minh rằng:
           1
            2
             1+
              +
                 1
                  3
                    + .... +
                             1
                              n
                                        (
                                > 2 n +1 −1             )
                                                   Với n là số nguyên

     Giải: Ta có
                     1
                          =
                      k 2 k
                             2
                                 >
                                       2
                                    k + k +1
                                             = 2 k +1 − k     (       )
     Khi cho k chạy từ 1 đến n ta có
     1 > 2 ( 2 − 1)
     1
      2
        >2 3− 2(        )
    ………………
     1
      n
               (
        > 2 n +1 − n            )
     Cộng từng vế các bất đẳng thức trên ta có 1 +
                                                                  1
                                                                   2
                                                                     +
                                                                       1
                                                                        3
                                                                          + .... +
                                                                                   1
                                                                                    n
                                                                                       (        )
                                                                                      > 2 n +1 −1
                                                n
                                                    1
     Ví dụ 3: Chứng minh rằng               ∑k
                                            k =1
                                                    2
                                                        <2          ∀n ∈ Z

                            1       1       1       1
     Giải: Ta có k 2 < k ( k − 1) = k − 1 − k
     Cho k chạy từ 2 đến n ta có
           1            1
             2
                < 1−
           2            2
           1 1 1
               < −
          32 2 3
          .................
           1         1       1
                <          −
           n 2
                   n −1 n
                1 1            1
          ⇒ 2 + 2 + .... + 2 < 1
               2 3             n

         n
               1
  Vậy   ∑k
        k =1
               2
                   <2



     Phương pháp 11: Dùng bất đẳng thức trong tam giác

     Kiến thức: Nếu a;b;clà số đo ba cạnh của tam giác thì : a;b;c> 0

Sưu tầm và tuyển chọn                                        16
19 Phương pháp chứng minh Bất đẳng thức_ Nguyễn Đức Hàn www.VNMATH.com
     Và |b-c| < a < b+c ; |a-c| < b < a+c ; |a-b| < c < b+a
     Ví dụ 1: Cho a;b;c là số đo ba cạnh của tam giác chứng minh rằng
     1/ a2+b2+c2< 2(ab+bc+ac)
     2/ abc>(a+b-c).(b+c-a).(c+a-b)
     Giải
     1/Vì a,b,c là số đo 3 cạnh của một tam giác nên ta có
            0 < a < b + c                                                 a 2 < a (b + c)
                                                                           2
            0 < b < a + c                ⇒                                b < b( a + c )
            0 < c < a + b                                                  c 2 < c ( a + b)
                                                                          
          Cộng từng vế các bất đẳng thức trên ta có: a2+b2+c2< 2(ab+bc+ac)
          2/ Ta có a > b-c  ⇒ a 2 > a 2 − (b − c) 2 > 0
              b > a-c     ⇒ b 2 > b 2 − (c − a ) 2 > 0
              c > a-b     ⇒ c 2 > c 2 − ( a − b) 2 > 0
          Nhân vế các bất đẳng thức ta được
                            [                    ][                   ][
             ⇒ a 2b 2 c 2 > a 2 − ( b − c ) b 2 − ( c − a ) c 2 − ( a − b )
                                             2                    2                        2
                                                                                               ]
             ⇒ a 2b 2 c 2 > ( a + b − c ) ( b + c − a ) ( c + a − b )
                                         2                2                     2


             ⇒ abc > ( a + b − c ).( b + c − a ).( c + a − b )

          Ví dụ2 (HS tự giải)
          1/ Cho a,b,c là chiều dài ba cạnh của tam giác
          Chứng minh rằng ab + bc + ca < a 2 + b 2 + c 2 < 2(ab + bc + ca)
          2/Cho a,b,c là chiều dài ba cạnh của tam giác có chu vi bằng 2
          Chứng minh rằng a 2 + b 2 + c 2 + 2abc < 2

          Phương pháp 12:                             Sử dụng hình học và tọa độ

          Ví dụ 1:
          Chứng minh rằng : c(a −c) + c(b −c) ≤ ab , ∀a ≥ b ≥ 0 và b ≥ c
          Giải
          Trong mặt phẳng Oxy, chọn u =( c, b −c ) ; v =( a −c , c )
          Thì u = b , v = a ;             u.v = c ( a −c ) + c (b −c )
          Hơn nữa: u.v =u . v . cos(u , v ) ≤u . v   ⇒ c ( a − ) + c (b − ) ≤ ab ⇒
                                                               c         c        (ĐPCM)
          Ví dụ 2:
                                                                                     n              n

          Cho 2n số:       xi ; y i , i =1,2,..., n           thỏa mãn: ∑ xi + ∑ yi = 1. Chứng minh rằng:
                                                                                    i =1           i =1
 n
                      2
∑
i =1
       xi2 + yi2 ≥
                     2
          Giải:
          Vẽ hình                                             y




                                                                                MN
                                                              MK
                                                                                     H
Sưu tầm và tuyển chọn                                                      17
19 Phương pháp chứng minh Bất đẳng thức_ Nguyễn Đức Hàn www.VNMATH.com

                                                              M
                                                              1
                                                                                                                      x
                                                          O                                            x+y=1



     Trong mặt phẳng tọa độ, xét:
      M 1 ( x1 , y1 ) : M 2 ( x1 + x 2 , y1 + y 2 ) ;…; M n ( x1 +  + x n , y1 +  + y n )

     Giả thiết suy ra                       M n ∈ đường thẳng x + y = 1. Lúc đó:
      OM 1 =           x12 + y12   ,        M 1M 2 =   x2 + y 2
                                                        2     2
                                                                  ,   M 2M 3 =   x3 + y 3
                                                                                  2     2
                                                                                            ,…,   M n −1 M n =       xn + y n
                                                                                                                      2     2


                                                                                                   2
     Và OM 1 + M 1 M 2 + M 2 M 3                        +  + M n −1 M n ≥ OM n ≥ OH =
                                                                                                  2
           n
                                        2
      ⇒   ∑
          i =1
                     xi2 + yi2 ≥
                                       2
                                              ⇒ (ĐPCM)




     Phương pháp 13:                             Đổi biến số
                                              a      b     c  3
     Ví dụ1: Cho a,b,c > 0 Chứng minh rằng        +     +    ≥ (1)
                                             b+c c+a a+b 2
                                               y+z−x         z+x− y      x+ y−z
     Giải: Đặt x=b+c ; y=c+a ;z= a+b ta có a=           ; b=        ;c=
                                                  2             2           2
                  y+z−x z+x− y x+ y−z          3
     ta có (1) ⇔          +        +         ≥
                     2x       2y      2z       2
           y z      x z      x y              y x      z x   z y
     ⇔       + −1+ + −1+ + −1 ≥ 3 ⇔ ( + ) + ( + ) + ( + ) ≥ 6
           x x      y y      z z              x y      x z   y z
                                       y x          z x      z y
     Bất đẳng thức cuối cùng đúng vì ( x + y ≥ 2;     + ≥ 2;  + ≥ 2 nên ta có điều
                                                             y z
                                                    x z
phải chứng minh
      Ví dụ2:
      Cho a,b,c > 0 và a+b+c <1. Chứng minh rằng
            1         1         1
                + 2        + 2      ≥9         (1)
         a + 2bc b + 2ac c + 2ab
                 2


     Giải: Đặt x = a 2 + 2bc ; y = b 2 + 2ac ; z = c 2 + 2ab . Ta có x + y + z = ( a + b + c ) 2 < 1
            1 1 1
     (1) ⇔ x + y + z ≥ 9       Với x+y+z < 1 và x ,y,z > 0
                                                               1 1 1             1
     Theo bất đẳng thức Côsi ta có: x + y + z ≥ 3. 3 xyz , và: x + y + z ≥ 3. 3
                                                                                xyz

                 ( x + y + z ). 1 + 1 + 1  ≥ 9 . Mà x+y+z < 1. Vậy
                                                                                            1 1 1
      ⇒                        
                               x y z                                                      + + ≥9              (đpcm)
                                                                                          x y z
                                                                                                                 1
     Ví dụ3: Cho x ≥ 0 , y ≥ 0 thỏa mãn 2 x − y = 1 CMR x + y ≥
                                                                                                                 5
     Gợi ý: Đặt x = u ,                            y =v     ⇒ 2u-v =1 và S = x+y = u 2 + v 2 ⇒ v = 2u-1


Sưu tầm và tuyển chọn                                                   18
19 Phương pháp chứng minh Bất đẳng thức_ Nguyễn Đức Hàn www.VNMATH.com
thay vào tính S min
      Bài tập tự giải
                                                             25a 16b   c
        1) Cho a > 0 , b > 0 , c > 0              CMR:          +    +   >8
                                                             b+c c+a a+b

        2)Tổng quát m, n, p, q, a, b >0
             CMR
                                    ma
                                       +
                                         nb
                                            +
                                              pc
                                    b+c c+a a+b 2
                                                 ≥
                                                   1
                                                       (              )
                                                           m + n + p − ( m + n + p)
                                                                          2




     Phương pháp 14:                            Dùng tam thức bậc hai

     Kiến thứ: Cho f(x) = ax2 + bx + c
     Định lí 1:
                           a> 0
       f(x) >     0, ∀ x ⇔ 
                           ∆ < 0
                        a> 0
       f (x) ≥ 0, ∀ x ⇔ 
                        ∆ ≤ 0
                        a< 0
       f (x) < 0, ∀ x ⇔ 
                        ∆ < 0
                        a< 0
       f (x) ≤ 0, ∀ x ⇔ 
                        ∆ ≤ 0
     Định lí 2:
     Phương trình f(x) = 0 có 2 nghiệm x1 < α < x2 ⇔ a. f ( α ) < 0
     Phương trình f(x) = 0 có 2 nghiệm :
                      
                       a. f ( α ) > 0
                      
        x1 < x2 < α ⇔  ∆ > 0
                      S
                       <α
                      2
     Phương trình f(x) = 0 có 2 nghiệm :



Sưu tầm và tuyển chọn                              19
19 Phương pháp chứng minh Bất đẳng thức_ Nguyễn Đức Hàn www.VNMATH.com
                       
                        a. f ( α ) > 0
                       
         α < x1 < x2 ⇔  ∆ > 0
                       S
                        >α
                       2
                                                                 α < x1 < β < x 2
     Phương trình f(x) = 0 có 2 nghiệm                            x < α < x < β ⇔ f (α ). f ( β ) < 0.
                                                                  1        2




     Ví dụ 1:Chứng minh rằng f ( x, y ) = x 2 + 5 y 2 − 4 xy + 2 x − 6 y + 3 > 0                                  (1)
     Giải: Ta có (1) ⇔ x 2 − 2 x( 2 y − 1) + 5 y 2 − 6 y + 3 > 0
       ∆′ = ( 2 y − 1) − 5 y 2 + 6 y − 3
                               2


          = 4 y2 − 4 y +1− 5y2 + 6 y − 3
        = −( y − 1) − 1 < 0
                                2


     Vậy f ( x, y ) > 0 với mọi x, y

     Ví dụ2:                       Chứng minh rằng: f ( x, y ) = x 2 y 4 + 2( x 2 + 2). y 2 + 4 xy + x 2 > 4 xy 3

     Giải: Bất đẳng thức cần chứng minh tương đương với
       x 2 y 4 + 2( x 2 + 2 ). y 2 + 4 xy + x 2 − 4 xy 3 > 0 ⇔ ( y 2 + 1) 2 .x 2 + 4 y (1 − y ) 2 x + 4 y 2 > 0
     Ta có ∆′ = 4 y 2 (1 − y 2 ) − 4 y 2 ( y 2 + 1) = −16 y 2 < 0
                                             2               2



     Vì a = ( y 2 + 1) > 0 vậy f ( x, y ) > 0
                      2
                                                              (đpcm)


       Phương pháp 15:          Dùng quy nạp toán học
       Kiến thức:
       Để chứng minh bất đẳng thức đúng với n > n0 ta thực hiện các bước sau :
       1 – Kiểm tra bất đẳng thức đúng với n = n0
       2 - Giả sử BĐT đúng với n =k (thay n =k vào BĐT cần chứng minh được gọi là
giả thiết quy nạp )
       3- Ta chứng minh bất đẳng thức đúng với n = k +1 (thay n = k+1vào BĐT cần
chứng minh rồi biến đổi để dùng giả thiết quy nạp)
       4 – kết luận BĐT đúng với mọi n > n0

                                  1 1              1       1
     Ví dụ1: Chứng minh rằng :     2
                                      + 2 + .... + 2 < 2 −        ∀n ∈ N ; n > 1                                        (1)
                                  1 2             n        n
                             1       1
     Giải: Với n =2 ta có 1 + < 2 −      (đúng). Vậy BĐT (1) đúng với n =2
                             4       2
     Giả sử BĐT (1) đúng với n =k ta phải chứng minh BĐT (1) đúng với n = k+1
                                                             1     1          1       1             1
     Thật vậy khi n =k+1 thì (1) ⇔ 12 + 22 + .... + k 2 + (k + 1) 2 < 2 − k + 1
     Theo giả thiết quy nạp


Sưu tầm và tuyển chọn                                              20
19 Phương pháp chứng minh Bất đẳng thức_ Nguyễn Đức Hàn www.VNMATH.com
            1 1             1      1          1     1             1
        ⇔      + 2 + .... + 2 +           < 2− +           < 2−
             2
            1 2            k    (k + 1) 2
                                              k ( k + 1) 2
                                                                k +1
            1              1          1       1        1
        ⇔      + .... +           <      +           <
            12
                        (k + 1) 2
                                    k + 1 ( k + 1) 2
                                                       k
             k +1+1 1
        ⇔             < ⇔ k (k + 2) < (k + 1) 2 ⇔ k2+2k<k2+2k+1                                        Điều này đúng .Vậy bất
             (k + 1) 2 k
đẳng thức (1)được chứng minh
                                                                                                           n
                                                                a+b   an + bn
        Ví dụ2: Cho           n ∈ N và a+b> 0. Chứng minh rằng      ≤         (1)
                                                                2        2
        Giải: Ta thấy BĐT (1) đúng với n=1
        Giả sử BĐT (1) đúng với n=k ta phải chứng minh BĐT đúng với n=k+1
        Thật vậy với n = k+1 ta có
                                   k +1
                    a+b                     a k +1 + b k +1
             (1) ⇔                      ≤
                    2                              2
                               k
                      a+b a+b              a k +1 + b k +1
               ⇔            .         ≤                    (2)
                    2             2              2
                          a k + b k a + b a k +1 + ab k + a k b + b k +1 a k +1 + b k +1
  ⇔ Vế trái (2) ≤                   .      =                              ≤
                              2         2                   4                     2
     k +1   k +1     k +1                  k +1
                                                 ≥ 0 ⇔ ( a k − b k ).( a − b ) ≥ 0
    a +b           a + ab + a b + b
                                k     k
  ⇔              −                                                                       (3)
          2                       4
        Ta chứng minh (3)
        (+) Giả sử a ≥ b và giả thiết cho a ≥ -b ⇔ a ≥ b
                               k
                  ⇔ a k ≥ b ≥ bk                ⇒      (a   k
                                                                − b k ).( a − b ) ≥ 0
        (+) Giả sử a < b và theo giả thiết - a<b ⇔ a < b k ⇔ a k < b k ⇔ ( a k − b k ).( a − b ) ≥ 0
                                                                                        k


        Vậy BĐT (3)luôn đúng ta có (đpcm)
        Ví dụ 3: Cho a ≥ −1 ,1 ≤ n ∈Ν. Chứng minh rằng : (1 + a) n ≥ 1 + n.a
        Giải
         n=1: bất đẳng thức luôn đúng
         n=k ( k ∈ Ν ): giả sử bất đẳng thức đúng, tức là: (1 + a ) k ≥ 1 + k .a
         n= k+1 . Ta cần chứng minh: (1 + a ) k +1 ≥ 1 + (k +1).a
        Ta có: (1 + a) k +1 = (1 + a).(1 + a) k ≥ (1 + a).(1 + k .a ) ≥ 1 + (k +1)a + k .a 2 ≥ 1 + (k +1)a
        ⇒ Bất đẳng thức đúng với n= k+1
        V ậy theo nguyên lý quy nạp: (1 + a) n ≥ 1 + n.a , ∀n ∈Ν
                                                                                                                    1
        Ví dụ 4: Cho 1 ≤ n ∈ Ν                a1 , a 2 ,  , a n ≥ 0      thoả mãn           a1 + a 2 +  + a n ≤     .   Chứng minh
                                                                                                                    2
                                               1
rằng:   (1 − a1 )(1 − a 2 )  (1 − a n ) ≥
                                               2

                              1           1
        Giải n=1:      a1 ≤     ⇒ 1 − a1 ≥ ⇒ Bài                   toán đúng
                              2           2
                                                                                                                                 1
        n=k ( k ∈ Ν ): giả sử bất đẳng thức đúng, tức là:                                   (1 − a1 )(1 − a 2 )  (1 − a k ) ≥
                                                                                                                                 2



Sưu tầm và tuyển chọn                                                  21
19 Phương pháp chứng minh Bất đẳng thức_ Nguyễn Đức Hàn www.VNMATH.com
                                                                                          1
           n= k+1 . Ta cần chứng minh:                    (1 − a1 )(1 − a 2 )  (1 − a k +1 ) ≥
                                                                                          2
           Ta có:   (1 − a1 )(1 − a 2 )  (1 − a k +1 ) = (1 − a1 )(1 − a 2 )  (1 − a k −1 )[1 − (a k + a k +1 ) + a k a k +1 ]
                                                                         1                                               1
           ≥ (1 − a1 )(1 − a 2 )  (1 − a k −1 )[1 − ( a k + a k +1 )] ≥   (Vì a1 + a 2 +  + a k −1 + (a k + a k +1 ) ≤ 2 )
                                                                         2
           ⇒ Bất đẳng thức đúng với n= k+1
                                                                                               1
           Vậy theo nguyên lý quy nạp:                    (1 − a1 )(1 − a 2 )  (1 − a n ) ≥
                                                                                               2
            Ví dụ 5: Cho 1 ≤ n ∈ Ν ,             a i , bi ∈R, i =1,2,..., n .    Chứng minh rằng:
            (a1b1 + a 2 b2 +  + a n bn ) ≤ (a + a +  + a )(b + b +  + bn2 )
                                          2     2
                                                1
                                                      2
                                                      2
                                                                  2
                                                                  n     1
                                                                         2   2
                                                                             2

           Giải n=1: Bất đẳng thức luôn đúng
           n=k ( k ∈ Ν ):giả sử bất đẳng thức đúng, tức là:
            (a1b1 + a 2 b2 +  + a k bk ) 2 ≤ (a12 + a 2 +  + a k )(b12 + b22 +  + bk2 )
                                                       2         2


            n= k+1 . Ta cần chứng minh:
            (a1b1 + a 2 b2 +  + a k +1bk +1 ) 2 ≤ (a12 + a 2 +  + a k +1 )(b12 + b2 +  + bk2+1 ) (1)
                                                            2         2             2


                      Thật vậy: VP(1) = (a12 + a 22 +  + a k2 )(b12 + b22 +  + bk2 ) + (a12 +  + a k2 ).b 2 +
           + a 2 (b12 + b2 +  + bk2 ) + a k +1 .bk2+1 ≥ ( a1b1 + a 2 b2 +  + a k bk ) + 2a1b1 a k +1bk +1 + 2a 2 b2 a k +1bk +1 +
                         2                 2


                   +  + 2a k bk a k +1bk +1 + a k +1bk +1
                                                 2    2


                   ≥ (a1b1 + a 2 b2 +  + a k bk ) 2 + 2 ( a1b1 + a 2 b2 +  + a k bk ) a k + bk + + a k +1 .bk +1
                                                                                             1    1
                                                                                                       2      2


                   ≥ (a1b1 + a 2 b2 +  + a k +1bk +1 ) 2
               Vậy (1) được chứng minh
           Ví dụ 6: Cho 1 ≤ n ∈ Ν , ai , bi ∈R, i =1,2,..., n . Chứng minh rằng:
             a1 + a 2 +  + a n 2 a12 + a 2 +  + a n
                                          2         2
           (                   ) ≤
                      n                     n
           Giải:
           n=1: Bất đẳng thức luôn đúng
                                                             a1 + a 2 +  + a k 2 a12 + a 2 +  + a k2
                                                                                          2

           n=k ( k ∈ Ν ):giả sử bất đẳng thức đúng, tức là: (                  ) ≤
                                                                      k                     k
                                        a + a +  + a k +1 2 a1 + a 2 +  + a k +1
                                                                2      2        2
           n= k+1 . Ta cần chứng minh: ( 1 2              ) ≤                      (1)
                                              k +1                      k +1
                   a + a +  + a k +1
           Đặt: a = 2 3
                          k
                          1
           VP (1) =          (a12 + k 2 a 2 + 2ka1 a )
                        k +1
       1         2       a 2 + a3 +  + a k2+1
                                 2
                                                            a 2 + a3 +  + a k2+1  a12 + a 2 +  + a k2+1
                                                                   2                        2
≥                a1 + k 2 2                    + k .a12 + k 2                    =
    (k + 1) 2                     k                                 k                      k +1
                  Vậy (1) đựơc chứng minh

           Ví dụ 7: Chứng minh rằng:                  n n > ( n +1) n −1 , ∀n ∈ Ζ, n ≥ 2

                        n n = 4
           Giải: n=2 ⇒                          ⇒ n n > (n + 1) n −1
                         (n + 1) n− 1 = 3
           n=k ≥ 2 : giả sử bất đẳng thức đúng, tức là: k k > (k + 1) k −1
           n= k+1:Ta c ó: k k (k + 1) k +1 ≥ (k + 1) k −1 (k + 1) k +1 = (k + 1) 2 k −2 (k + 1) 2         = [(k + 1) 2 ] k −1 (k + 1) 2

Sưu tầm và tuyển chọn                                                 22
19 Phương pháp chứng minh Bất đẳng thức_ Nguyễn Đức Hàn www.VNMATH.com
     > (k 2 + 2k ) k −1 ( k 2 + 2k ) (vì (k + 1) 2 = k 2 + 2k + 1 > k 2 + 2k )
     ≥ k k ( k + 2) k ⇒ (k + 1) k +1 > (k + 2) k ⇒ Bất đẳng thức đúng với n= k+1
     Vậy n n > (n +1) n−1 , ∀n ∈ Ζ, n ≥ 2
     Ví dụ 8: Chứng minh rằng: sin nx ≤n sin x , ∀ ∈Ν , ∀ ∈R       ∗
                                                                        n      x

     Giải: n=1: Bất đẳng thức luôn đúng
     n=k :giả sử bất đẳng thức đúng, tức là: sin kx ≤k sin x
     n= k+1 . Ta cần chứng minh: sin(k + ) x ≤(k + ) sin x  1             1


              a + b ≤ a + b , ∀ a, b ∈ R
             
      Ta có: 
              sin x , cos x ≤ 1, ∀ x ∈ R
             
      Nên:   sin( k +1) x = sin kx cos x +cos kx sin x
       ≤ sin kx . cos x + cos kx . sin x ≤ sin kx . + sin x ≤k sin x . +. sin x =( k + ) sin x
                                                     .                                1
       ⇒ Bất đẳng thức đúng với n= k+1. Vậy:             sin nx ≤n sin x , ∀ ∈ ∗, ∀ ∈
                                                                            n Ν    x R




      Phương pháp 16:           Chứng minh phản chứng

       Kiến thức:
       1) Giả sử phải chứng minh bất đẳng thức nào đó đúng , ta hãy giả sử bất đẳng
thức đó sai và kết hợp với các giả thiết để suy ra điều vô lý , điều vô lý có thể là điều trái
với giả thiết , có thể là điều trái ngược nhau .Từ đó suy ra bất đẳng thức cần chứng minh
là đúng
       2) Giả sử ta phải chứng minh luận đề “p ⇒ q”
       Muốn chứng minh p ⇒ q (với p : giả thiết đúng, q : kết luận đúng) phép chứng
minh được thực hiên như sau:
       Giả sử không có q ( hoặc q sai) suy ra điều vô lý hoặc p sai. Vậy phải có q
(hay q đúng)
       Như vậy để phủ định luận đề ta ghép tất cả giả thiết của luận đề với phủ định kết
luận của nó .
    Ta thường dùng 5 hình thức chứng minh phản chứng sau :
     A - Dùng mệnh đề phản đảo : “P ⇒ Q”
     B – Phủ định rôi suy trái giả thiết
     C – Phủ định rồi suy trái với điều đúng
     D – Phủ định rồi suy ra 2 điều trái ngược nhau
     E – Phủ định rồi suy ra kết luận :


      Ví dụ 1: Cho ba số a,b,c thỏa mãn a +b+c > 0 , ab+bc+ac > 0 , abc > 0
      Chứng minh rằng a > 0 , b > 0 , c > 0
      Giải:
      Giả sử a ≤ 0 thì từ abc > 0 ⇒ a ≠ 0 do đó a < 0. Mà abc > 0 và a < 0 ⇒ cb < 0
     Từ ab+bc+ca > 0 ⇒ a(b+c) > -bc > 0
      Vì a < 0 mà a(b +c) > 0 ⇒ b + c < 0
     a < 0 và b +c < 0 ⇒ a + b +c < 0 trái giả thiết a+b+c > 0

Sưu tầm và tuyển chọn                             23
19 Phương pháp chứng minh Bất đẳng thức_ Nguyễn Đức Hàn www.VNMATH.com
     Vậy a > 0 tương tự ta có b > 0 , c > 0
      Ví dụ 2:Cho 4 số a , b , c ,d thỏa mãn điều kiện
     ac ≥ 2.(b+d) .Chứng minh rằng có ít nhất một trong các bất đẳng thức sau là sai:
                   a 2 < 4b , c 2 < 4d
      Giải:
      Giả sử 2 bất đẳng thức : a 2 < 4b , c 2 < 4d đều đúng khi đó cộng các vế ta được
      a 2 + c 2 < 4(b + d ) (1)
      Theo giả thiết ta có 4(b+d) ≤ 2ac (2)
      Từ (1) và (2) ⇒ a 2 + c 2 < 2ac hay ( a − c ) 2 < 0 (vô lý)
      Vậy trong 2 bất đẳng thức a 2 < 4b và c 2 < 4d có ít nhất một các bất đẳng thức sai
      Ví dụ 3:Cho x,y,z > 0 và xyz = 1. Chứng minh rằng
                           1     1   1
      Nếu x+y+z > x + y + z thì có một trong ba số này lớn hơn 1
      Giải :Ta có (x-1).(y-1).(z-1) =xyz – xy- yz + x + y+ z –1
                           1     1   1                                            1      1   1
      =x + y + z – ( x + y + z ) vì xyz = theo giả thiết x+y +z > x + y + z
nên (x-1).(y-1).(z-1) > 0
      Trong ba số x-1 , y-1 , z-1 chỉ có một số dương
      Thật vậy nếu cả ba số dương thì x,y,z > 1 ⇒ xyz > 1 (trái giả thiết)
      Còn nếu 2 trong 3 số đó dương thì (x-1).(y-1).(z-1) < 0 (vô lý)
      Vậy có một và chỉ một trong ba số x , y,z lớn hơn 1
      Ví dụ 4: Cho a, b, c >0 và a.b.c=1. Chứng minh rằng: a + b + c ≥ 3 (Bất đẳng thức
Cauchy 3 số)
      Giải: Giả sử ngược l ại:
      a + b + c < 3 ⇒ (a + b + c)ab < 3ab ⇔ a 2 b + b 2 a + cab < 3ab ⇔ a 2 b + (a 2 − 3a )b + 1 < 0
      Xét : f (b) = a 2 b + (a 2 − 3a)b + 1
      Có ∆ = (a 2 − 3a) 2 − 4a = a 4 − 6a 3 + 9a 2 − 4a = a(a 3 − 6a 2 + 9a − 4) = = a(a −1) 2 (a − 4) ≤ 0
           a , b, c > 0
      (Vì                     ⇒ 0 < a < 3 ) ⇒ f (b) ≥ 0 ⇒ vô   lý. Vậy: a + b + c ≥ 3
          a+ b+ c< 3
      Ví dụ 5:
      Chứng minh rằng không tồn tại các số a, b, c đồng thời thỏa mãn (1),(2),(3):
      a < b −c       (1)
      b < c −a     (2)
      c < a −b     (3)
      Giải: Giả sử tồn tại các số a, b, c đồng thời thỏa mãn (1),(2),(3), lúc đó:
      a < b −c ⇒ (b − c ) 2 > a 2    ⇒ −( a + b − c )(a − b + c) > 0                  (1’)
      b < c −a    ⇒ (c − a ) > b
                             2     2
                                     ⇒ −(−a + b + c)(a + b − c ) > 0                         (2’)
      c < a −b    ⇒ ( a − b) 2 > c 2  ⇒ −(a + b − c)(−a + b + c) > 0             (3’)
      Nhân (1’), (2’) và (3’) vế với vế ta được:         ⇒ −[(a + b − c)( a − b + c)(−a + b + c)] 2 > 0
      ⇒ Vô lý. Vậy bài toán được chứng minh


      Phương pháp 17 :                   Sử dụng biến đổi lượng giác
Sưu tầm và tuyển chọn                                24
19 Phương pháp chứng minh Bất đẳng thức_ Nguyễn Đức Hàn www.VNMATH.com

                                                                                                                 − π π 
     1. Nếu      x ≤R      thì đặt x = Rcos α ,                    α ∈ [ 0, π ] ; hoặc x = Rsin α ,           α ∈   ,
                                                                                                                  2 2 
                                                   R                                     π
     2. Nếu      x ≥R      thì đặt x =                         α ∈[0, c ) ∪ π,3              
                                                 cos α                                      2


                                                                      x = a + R cosα
     3.Nếu ( x − a ) 2 + ( y − b ) 2    = R 2 , ( > 0)       thì đặt                  , (α = 2π )
                                                                      y = b + R sinα
                                                                               x = α + aR cosα
                                                                                               , (α = 2π )
                           2                 2
                 x −α    y −β 
     4. Nếu            +       =R
                                      2
                                                       a, b > 0 thì đặt

                                                                               y = β + bR sinα
                 a       b 


     5. Nếu trong bài toán xuất hiện biểu thức : ( ax ) 2 + b 2 , ( a, b > 0)
                       b            π π
     Thì đặt:     x=     tgα , α ∈  − , 
                       a            2 2


     Ví dụ 1: Cmr : a 1 −b               2
                                             +b 1 −a 2 + 3 ab −      (           (1 −b )(1 −a ) ) ≤2, ∀ , b ∈[−1,1]
                                                                                         2          2
                                                                                                       a

     Giải : a ≤1, b ≤1
              a = cosα
     Đặt    :                       (α , β ∈ [ 0, π ] )
              b = cos β
     Khi đó :
      a 1 − b 2 + b 1 − a 2 + 3 ab −    (         (1 − b )(1 − a ) )
                                                           2         2


          = cos α. sin β + cos β. sin α + 3 ( cos α. cos β − sin α. sin β )
          = sin(α + β ) + 3. cos(α + β )
                                π
          = 2 cos(α + β −          ) ∈[ − 2,2] ⇒ ( dpcm)
                                 6
     Ví dụ 2 : Cho             a , b ≥1 .Chứng minh               rằng : a        b −1 + b a1 ≤ ab
     Giải :
                  1
            a = cos 2 α
                                           π 
     Đặt :        1               α , β ∈  0,  
                                                  
           b =                             2
            cos 2 β
           
                                       1              1
      ⇒ a b −1 + b a −1 =                  tg 2 β +                      tg 2α
                                     cos α
                                        2
                                                    cos 2 β
         tgβ        tgα      (tgβ . cos 2 β + tgα . cos 2 α )
      =         +          =
        cos 2 α cos 2 β              cos 2 β . cos 2 α
        1 (sin 2 β + sin 2α ) sin(α + β ) cos in(α − β )
      =                       =
        2 cos 2 β . cos 2 α           cos 2 β . cos 2 α
               1
      ≤                  = ab
        cos β . cos 2 α
            2




Sưu tầm và tuyển chọn                                               25
19 Phương pháp chứng minh Bất đẳng thức_ Nguyễn Đức Hàn www.VNMATH.com
                                                         a 2 − (a − 4b) 2
      Ví dụ 3: Cho ab ≠ 0 .Chứng minh rằng : − 2 2 − 2 ≤                  ≤2 2−2
                                                              2      2
                                                                               a + 4b
      Giải :
                                                     a 2 − (a − 4b) 2 tg 2α − (tgα − 2) 2
                                                 ⇒                   =
                                                         a 2 + 4b 2        1 + tg 2α
                                                                = 4(tgα − 1). cos 2 α
                                π π 
          Đặt: a = 2btgα , α ∈  − 2 , 22                      = 2 sin 2α − 2(1 + cos 2α )
                                         
                                                                = 2(sin 2α − cos 2α ) − 2
                                                                                 π
                                                                                            [
                                                                = 2 2 sin( 2α − ) − 2 ∈ − 2 2 − 2,2 2 − 2
                                                                                  2
                                                                                                            ]
      Phương pháp 18:        Sử dụng khai triển nhị thức Newton.
      Kiến thức:
      Công thức nhị thức Newton
                 n
      ( a + b ) n ∑C nk a n −k b k , ∀n ∈ N * , ∀a, b ∈ R .
                k =0
                                           n!
      Trong đó hệ số C n
                                  k
                                      =                       (0 ≤ ≤ )
                                                                  k n          .
                                       ( n − )! k!
                                            k
      Một số tính chất đặt biệt của khai triển nhị thức Newton:
      + Trong khai triển (a + b)n có n + 1 số hạng.
      + Số mũ của a giảm dần từ n đến 0, trong khi đó số mũ của b tăng từ 0 đến n.
Trong mỗi số hạng của khai trtiển nhị thức Newton có tổng số mũ của a và b bằng n.
      +Các hệ số cách đều hai đầu thì bằng nhau
       C n = C n −k .
         k     n


      + Số hạng thứ k + 1 là C nk a n −k .b k (0 ≤ k ≤ n)
      Ví dụ 1:
      Chứng minh rằng (1 + a ) n ≥ 1 + na, ∀a ≥ 0, ∀n ∈ N * (bất đẳng thức bernoulli)
      Giải
                            n

      Ta có: (1 + a ) = ∑ C n a ≥ C n + C n a = 1 + na (đpcm)
                     n      k k     0     1

                           k =0

      Ví dụ 2:
      Chứng minh rằng:
                                  n
        an +bn   a +b
      a) 2 ≥  2  , ∀a, b ≥ 0, ∀n ∈ N *
                     
                     
                                         n
         an +bn + cn  a +b + c 
      b)            ≥           , ∀a, b, c ≥ 0, ∀n ∈ N
                                                         *

             3           3     
      Giải
      Theo công thức khai triển nhị thức Newton ta có:




Sưu tầm và tuyển chọn                                   26
19 phương phap chứng minh bất đẳng thức
19 phương phap chứng minh bất đẳng thức
19 phương phap chứng minh bất đẳng thức
19 phương phap chứng minh bất đẳng thức
19 phương phap chứng minh bất đẳng thức
19 phương phap chứng minh bất đẳng thức
19 phương phap chứng minh bất đẳng thức
19 phương phap chứng minh bất đẳng thức
19 phương phap chứng minh bất đẳng thức
19 phương phap chứng minh bất đẳng thức
19 phương phap chứng minh bất đẳng thức

Contenu connexe

Tendances

Bài toán liên quan về Phân số tối giản trong Toán lớp 6
Bài toán liên quan về Phân số tối giản trong Toán lớp 6Bài toán liên quan về Phân số tối giản trong Toán lớp 6
Bài toán liên quan về Phân số tối giản trong Toán lớp 6Bồi dưỡng Toán lớp 6
 
Chứng minh bất đẳng thức bằng phương pháp chọn điểm rơi. (1)
Chứng minh bất đẳng thức bằng phương pháp chọn điểm rơi. (1)Chứng minh bất đẳng thức bằng phương pháp chọn điểm rơi. (1)
Chứng minh bất đẳng thức bằng phương pháp chọn điểm rơi. (1)Sao Băng Lạnh Giá
 
đại số tuyến tính 2 ( không gian eculid )
đại số tuyến tính 2 ( không gian eculid )đại số tuyến tính 2 ( không gian eculid )
đại số tuyến tính 2 ( không gian eculid )Bui Loi
 
BĐT Côsi ngược dấu
BĐT Côsi ngược dấuBĐT Côsi ngược dấu
BĐT Côsi ngược dấunhankhangvt
 
Tuyen tap-400-bai-bat-dang-thuc-co-giai-chi-tiet
Tuyen tap-400-bai-bat-dang-thuc-co-giai-chi-tietTuyen tap-400-bai-bat-dang-thuc-co-giai-chi-tiet
Tuyen tap-400-bai-bat-dang-thuc-co-giai-chi-tietToán THCS
 
Chuyen de toan logic roi rac li thuyet to hop
Chuyen de toan logic  roi rac li thuyet to hopChuyen de toan logic  roi rac li thuyet to hop
Chuyen de toan logic roi rac li thuyet to hoplephucduc06011999
 
Chuyên đề Đẳng Thức và Bất đẳng thức - Bồi dưỡng HSG môn Toán lớp 9
Chuyên đề Đẳng Thức và Bất đẳng thức - Bồi dưỡng HSG môn Toán lớp 9Chuyên đề Đẳng Thức và Bất đẳng thức - Bồi dưỡng HSG môn Toán lớp 9
Chuyên đề Đẳng Thức và Bất đẳng thức - Bồi dưỡng HSG môn Toán lớp 9BOIDUONGTOAN.COM
 
Nhị thức newton và Phương pháp giải các bài tập về Nhị thức newton
Nhị thức newton và Phương pháp giải các bài tập về Nhị thức newtonNhị thức newton và Phương pháp giải các bài tập về Nhị thức newton
Nhị thức newton và Phương pháp giải các bài tập về Nhị thức newtonLinh Nguyễn
 
Kỹ thuật nhân liên hợp
Kỹ thuật nhân liên hợpKỹ thuật nhân liên hợp
Kỹ thuật nhân liên hợptuituhoc
 
13 ki-thuat-giai-phuong-trinh-ham (1)
13 ki-thuat-giai-phuong-trinh-ham (1)13 ki-thuat-giai-phuong-trinh-ham (1)
13 ki-thuat-giai-phuong-trinh-ham (1)ljmonking
 
Hệ Hoán Vị Vòng Quanh
Hệ Hoán Vị Vòng QuanhHệ Hoán Vị Vòng Quanh
Hệ Hoán Vị Vòng QuanhNhập Vân Long
 
Tuyển tập các bài Toán Hình học lớp 9 ôn thi vào 10
Tuyển tập các bài Toán Hình học lớp 9 ôn thi vào 10Tuyển tập các bài Toán Hình học lớp 9 ôn thi vào 10
Tuyển tập các bài Toán Hình học lớp 9 ôn thi vào 10BOIDUONGTOAN.COM
 
Gt khong gian_metric Nguyen Hoang
Gt khong gian_metric Nguyen HoangGt khong gian_metric Nguyen Hoang
Gt khong gian_metric Nguyen HoangBui Loi
 
Cong thuc luong giac day du
Cong thuc luong giac  day duCong thuc luong giac  day du
Cong thuc luong giac day duLe Nguyen
 
72 hệ phương trình
72 hệ phương trình72 hệ phương trình
72 hệ phương trìnhHades0510
 

Tendances (20)

Bài toán liên quan về Phân số tối giản trong Toán lớp 6
Bài toán liên quan về Phân số tối giản trong Toán lớp 6Bài toán liên quan về Phân số tối giản trong Toán lớp 6
Bài toán liên quan về Phân số tối giản trong Toán lớp 6
 
Chứng minh bất đẳng thức bằng phương pháp chọn điểm rơi. (1)
Chứng minh bất đẳng thức bằng phương pháp chọn điểm rơi. (1)Chứng minh bất đẳng thức bằng phương pháp chọn điểm rơi. (1)
Chứng minh bất đẳng thức bằng phương pháp chọn điểm rơi. (1)
 
đại số tuyến tính 2 ( không gian eculid )
đại số tuyến tính 2 ( không gian eculid )đại số tuyến tính 2 ( không gian eculid )
đại số tuyến tính 2 ( không gian eculid )
 
BĐT Côsi ngược dấu
BĐT Côsi ngược dấuBĐT Côsi ngược dấu
BĐT Côsi ngược dấu
 
Tuyen tap-400-bai-bat-dang-thuc-co-giai-chi-tiet
Tuyen tap-400-bai-bat-dang-thuc-co-giai-chi-tietTuyen tap-400-bai-bat-dang-thuc-co-giai-chi-tiet
Tuyen tap-400-bai-bat-dang-thuc-co-giai-chi-tiet
 
Chuyen de toan logic roi rac li thuyet to hop
Chuyen de toan logic  roi rac li thuyet to hopChuyen de toan logic  roi rac li thuyet to hop
Chuyen de toan logic roi rac li thuyet to hop
 
Chuyên đề Đẳng Thức và Bất đẳng thức - Bồi dưỡng HSG môn Toán lớp 9
Chuyên đề Đẳng Thức và Bất đẳng thức - Bồi dưỡng HSG môn Toán lớp 9Chuyên đề Đẳng Thức và Bất đẳng thức - Bồi dưỡng HSG môn Toán lớp 9
Chuyên đề Đẳng Thức và Bất đẳng thức - Bồi dưỡng HSG môn Toán lớp 9
 
Nhị thức newton và Phương pháp giải các bài tập về Nhị thức newton
Nhị thức newton và Phương pháp giải các bài tập về Nhị thức newtonNhị thức newton và Phương pháp giải các bài tập về Nhị thức newton
Nhị thức newton và Phương pháp giải các bài tập về Nhị thức newton
 
Kỹ thuật nhân liên hợp
Kỹ thuật nhân liên hợpKỹ thuật nhân liên hợp
Kỹ thuật nhân liên hợp
 
13 ki-thuat-giai-phuong-trinh-ham (1)
13 ki-thuat-giai-phuong-trinh-ham (1)13 ki-thuat-giai-phuong-trinh-ham (1)
13 ki-thuat-giai-phuong-trinh-ham (1)
 
Hệ Hoán Vị Vòng Quanh
Hệ Hoán Vị Vòng QuanhHệ Hoán Vị Vòng Quanh
Hệ Hoán Vị Vòng Quanh
 
Luận văn: Phép biến đổi phân tuyến tính, HAY, 9đ
Luận văn: Phép biến đổi phân tuyến tính, HAY, 9đLuận văn: Phép biến đổi phân tuyến tính, HAY, 9đ
Luận văn: Phép biến đổi phân tuyến tính, HAY, 9đ
 
Bt dai so hoang
Bt dai so hoangBt dai so hoang
Bt dai so hoang
 
Scp mod p
Scp mod pScp mod p
Scp mod p
 
Bất đẳng thức hình học
Bất đẳng thức hình họcBất đẳng thức hình học
Bất đẳng thức hình học
 
Tuyển tập các bài Toán Hình học lớp 9 ôn thi vào 10
Tuyển tập các bài Toán Hình học lớp 9 ôn thi vào 10Tuyển tập các bài Toán Hình học lớp 9 ôn thi vào 10
Tuyển tập các bài Toán Hình học lớp 9 ôn thi vào 10
 
Gt khong gian_metric Nguyen Hoang
Gt khong gian_metric Nguyen HoangGt khong gian_metric Nguyen Hoang
Gt khong gian_metric Nguyen Hoang
 
Cong thuc luong giac day du
Cong thuc luong giac  day duCong thuc luong giac  day du
Cong thuc luong giac day du
 
72 hệ phương trình
72 hệ phương trình72 hệ phương trình
72 hệ phương trình
 
Dãy số tuyến tính
Dãy số tuyến tínhDãy số tuyến tính
Dãy số tuyến tính
 

Similaire à 19 phương phap chứng minh bất đẳng thức

19 phương pháp chứng minh bđt
19 phương pháp chứng minh bđt19 phương pháp chứng minh bđt
19 phương pháp chứng minh bđtCảnh
 
19 phuong phap chung minh bat dang thu ccb
 19 phuong phap chung minh bat dang thu ccb 19 phuong phap chung minh bat dang thu ccb
19 phuong phap chung minh bat dang thu ccbPTAnh SuperA
 
Tuyen tap cac_bat_dang_thuc_trong_cac_de_thi_tuyen_sing_dai_hoc(ca_hd)
Tuyen tap cac_bat_dang_thuc_trong_cac_de_thi_tuyen_sing_dai_hoc(ca_hd)Tuyen tap cac_bat_dang_thuc_trong_cac_de_thi_tuyen_sing_dai_hoc(ca_hd)
Tuyen tap cac_bat_dang_thuc_trong_cac_de_thi_tuyen_sing_dai_hoc(ca_hd)Nguyen KienHuyen
 
Da toan-chi-tiet-b 2010
Da toan-chi-tiet-b 2010Da toan-chi-tiet-b 2010
Da toan-chi-tiet-b 2010nhathung
 
Thi thử toán mai thúc loan ht 2012 lần 1
Thi thử toán mai thúc loan ht 2012 lần 1Thi thử toán mai thúc loan ht 2012 lần 1
Thi thử toán mai thúc loan ht 2012 lần 1Thế Giới Tinh Hoa
 
Bdt của tran si tung
Bdt của tran si tungBdt của tran si tung
Bdt của tran si tungCam huynh
 
270 bai toan_boi_duong_hs_gioi_va_nang_khieu_toan
270 bai toan_boi_duong_hs_gioi_va_nang_khieu_toan270 bai toan_boi_duong_hs_gioi_va_nang_khieu_toan
270 bai toan_boi_duong_hs_gioi_va_nang_khieu_toanTam Vu Minh
 
270 bai toan bdhsg 9doc
270  bai toan bdhsg 9doc270  bai toan bdhsg 9doc
270 bai toan bdhsg 9docTam Vu Minh
 
6856978 collected-problems-about-inequality
6856978 collected-problems-about-inequality6856978 collected-problems-about-inequality
6856978 collected-problems-about-inequalityria_nghia
 
Thi thử toán vmf 2012 lần 3 đáp án
Thi thử toán vmf 2012 lần 3 đáp ánThi thử toán vmf 2012 lần 3 đáp án
Thi thử toán vmf 2012 lần 3 đáp ánThế Giới Tinh Hoa
 
Chuyên đề phương trình chứa căn thức bookbooming
Chuyên đề phương trình chứa căn thức   bookboomingChuyên đề phương trình chứa căn thức   bookbooming
Chuyên đề phương trình chứa căn thức bookboomingThế Giới Tinh Hoa
 
Toanvao10 2011
Toanvao10 2011Toanvao10 2011
Toanvao10 2011Duy Duy
 
[Www.toan trunghoccoso.toancapba.net] toan 9 chuyen de bd hsg nang khieu
[Www.toan trunghoccoso.toancapba.net] toan 9  chuyen de bd hsg  nang khieu[Www.toan trunghoccoso.toancapba.net] toan 9  chuyen de bd hsg  nang khieu
[Www.toan trunghoccoso.toancapba.net] toan 9 chuyen de bd hsg nang khieuTam Vu Minh
 
Thi thử toán thuận thành 1 bn 2012 lần 2
Thi thử toán thuận thành 1 bn 2012 lần 2Thi thử toán thuận thành 1 bn 2012 lần 2
Thi thử toán thuận thành 1 bn 2012 lần 2Thế Giới Tinh Hoa
 
Da2010 day-du 2010
Da2010 day-du 2010Da2010 day-du 2010
Da2010 day-du 2010nhathung
 

Similaire à 19 phương phap chứng minh bất đẳng thức (20)

19 phương pháp chứng minh bđt
19 phương pháp chứng minh bđt19 phương pháp chứng minh bđt
19 phương pháp chứng minh bđt
 
19 phuong phap chung minh bat dang thu ccb
 19 phuong phap chung minh bat dang thu ccb 19 phuong phap chung minh bat dang thu ccb
19 phuong phap chung minh bat dang thu ccb
 
Tuyen tap cac_bat_dang_thuc_trong_cac_de_thi_tuyen_sing_dai_hoc(ca_hd)
Tuyen tap cac_bat_dang_thuc_trong_cac_de_thi_tuyen_sing_dai_hoc(ca_hd)Tuyen tap cac_bat_dang_thuc_trong_cac_de_thi_tuyen_sing_dai_hoc(ca_hd)
Tuyen tap cac_bat_dang_thuc_trong_cac_de_thi_tuyen_sing_dai_hoc(ca_hd)
 
Da toan-chi-tiet-b 2010
Da toan-chi-tiet-b 2010Da toan-chi-tiet-b 2010
Da toan-chi-tiet-b 2010
 
Thi thử toán mai thúc loan ht 2012 lần 1
Thi thử toán mai thúc loan ht 2012 lần 1Thi thử toán mai thúc loan ht 2012 lần 1
Thi thử toán mai thúc loan ht 2012 lần 1
 
Bdt của tran si tung
Bdt của tran si tungBdt của tran si tung
Bdt của tran si tung
 
Bdt võ quốc bá cẩn
Bdt  võ quốc bá cẩnBdt  võ quốc bá cẩn
Bdt võ quốc bá cẩn
 
500 bdt
500 bdt500 bdt
500 bdt
 
270 bai toan_boi_duong_hs_gioi_va_nang_khieu_toan
270 bai toan_boi_duong_hs_gioi_va_nang_khieu_toan270 bai toan_boi_duong_hs_gioi_va_nang_khieu_toan
270 bai toan_boi_duong_hs_gioi_va_nang_khieu_toan
 
270 bai toan bdhsg 9doc
270  bai toan bdhsg 9doc270  bai toan bdhsg 9doc
270 bai toan bdhsg 9doc
 
6856978 collected-problems-about-inequality
6856978 collected-problems-about-inequality6856978 collected-problems-about-inequality
6856978 collected-problems-about-inequality
 
Bài 1.thidh-autosaved
Bài 1.thidh-autosavedBài 1.thidh-autosaved
Bài 1.thidh-autosaved
 
Thi thử toán vmf 2012 lần 3 đáp án
Thi thử toán vmf 2012 lần 3 đáp ánThi thử toán vmf 2012 lần 3 đáp án
Thi thử toán vmf 2012 lần 3 đáp án
 
Chuyên đề phương trình chứa căn thức bookbooming
Chuyên đề phương trình chứa căn thức   bookboomingChuyên đề phương trình chứa căn thức   bookbooming
Chuyên đề phương trình chứa căn thức bookbooming
 
Toanvao10 2011
Toanvao10 2011Toanvao10 2011
Toanvao10 2011
 
[Www.toan trunghoccoso.toancapba.net] toan 9 chuyen de bd hsg nang khieu
[Www.toan trunghoccoso.toancapba.net] toan 9  chuyen de bd hsg  nang khieu[Www.toan trunghoccoso.toancapba.net] toan 9  chuyen de bd hsg  nang khieu
[Www.toan trunghoccoso.toancapba.net] toan 9 chuyen de bd hsg nang khieu
 
Thi thử toán thuận thành 1 bn 2012 lần 2
Thi thử toán thuận thành 1 bn 2012 lần 2Thi thử toán thuận thành 1 bn 2012 lần 2
Thi thử toán thuận thành 1 bn 2012 lần 2
 
Da2010 day-du 2010
Da2010 day-du 2010Da2010 day-du 2010
Da2010 day-du 2010
 
Basic số phức cực hay
Basic số phức cực hayBasic số phức cực hay
Basic số phức cực hay
 
Tuyển tập 300 bài bdt
Tuyển tập 300 bài bdtTuyển tập 300 bài bdt
Tuyển tập 300 bài bdt
 

Plus de Thế Giới Tinh Hoa

Cách chụp ảnh công ty đẹp 2019
Cách chụp ảnh công ty đẹp 2019Cách chụp ảnh công ty đẹp 2019
Cách chụp ảnh công ty đẹp 2019Thế Giới Tinh Hoa
 
Bảng báo giá sản phẩm rèm bạch dương
Bảng báo giá sản phẩm rèm bạch dươngBảng báo giá sản phẩm rèm bạch dương
Bảng báo giá sản phẩm rèm bạch dươngThế Giới Tinh Hoa
 
Album sổ mẫu Rèm cửa Bạch Dương
Album sổ mẫu Rèm cửa Bạch DươngAlbum sổ mẫu Rèm cửa Bạch Dương
Album sổ mẫu Rèm cửa Bạch DươngThế Giới Tinh Hoa
 
Cách tắm cho bé vào mùa đông
Cách tắm cho bé vào mùa đôngCách tắm cho bé vào mùa đông
Cách tắm cho bé vào mùa đôngThế Giới Tinh Hoa
 
Giáo trình tự học illustrator cs6
Giáo trình tự học illustrator cs6  Giáo trình tự học illustrator cs6
Giáo trình tự học illustrator cs6 Thế Giới Tinh Hoa
 
Nữ quái sân trườngtruonghocso.com
Nữ quái sân trườngtruonghocso.comNữ quái sân trườngtruonghocso.com
Nữ quái sân trườngtruonghocso.comThế Giới Tinh Hoa
 
Những chàng trai xấu tính nguyễn nhật ánhtruonghocso.com
Những chàng trai xấu tính  nguyễn nhật ánhtruonghocso.comNhững chàng trai xấu tính  nguyễn nhật ánhtruonghocso.com
Những chàng trai xấu tính nguyễn nhật ánhtruonghocso.comThế Giới Tinh Hoa
 
Những bài văn mẫu dành cho học sinh lớp 10truonghocso.com
Những bài văn mẫu dành cho học sinh lớp 10truonghocso.comNhững bài văn mẫu dành cho học sinh lớp 10truonghocso.com
Những bài văn mẫu dành cho học sinh lớp 10truonghocso.comThế Giới Tinh Hoa
 

Plus de Thế Giới Tinh Hoa (20)

Cách chụp ảnh công ty đẹp 2019
Cách chụp ảnh công ty đẹp 2019Cách chụp ảnh công ty đẹp 2019
Cách chụp ảnh công ty đẹp 2019
 
Lỗi web bachawater
Lỗi web bachawaterLỗi web bachawater
Lỗi web bachawater
 
Bảng báo giá sản phẩm rèm bạch dương
Bảng báo giá sản phẩm rèm bạch dươngBảng báo giá sản phẩm rèm bạch dương
Bảng báo giá sản phẩm rèm bạch dương
 
Album sổ mẫu Rèm cửa Bạch Dương
Album sổ mẫu Rèm cửa Bạch DươngAlbum sổ mẫu Rèm cửa Bạch Dương
Album sổ mẫu Rèm cửa Bạch Dương
 
thong tin lam viec tren lamchame
thong tin lam viec tren lamchamethong tin lam viec tren lamchame
thong tin lam viec tren lamchame
 
Cách tắm cho bé vào mùa đông
Cách tắm cho bé vào mùa đôngCách tắm cho bé vào mùa đông
Cách tắm cho bé vào mùa đông
 
Giáo trình tự học illustrator cs6
Giáo trình tự học illustrator cs6  Giáo trình tự học illustrator cs6
Giáo trình tự học illustrator cs6
 
Nang luc truyen thong
Nang luc truyen thongNang luc truyen thong
Nang luc truyen thong
 
Huongdansudung izishop
Huongdansudung izishopHuongdansudung izishop
Huongdansudung izishop
 
Ho so nang luc cong ty
Ho so nang luc cong tyHo so nang luc cong ty
Ho so nang luc cong ty
 
seo contract
seo contractseo contract
seo contract
 
di google cong
di google congdi google cong
di google cong
 
E1 f4 bộ binh
E1 f4 bộ binhE1 f4 bộ binh
E1 f4 bộ binh
 
E2 f2 bộ binh
E2 f2 bộ binhE2 f2 bộ binh
E2 f2 bộ binh
 
E3 f1 bộ binh
E3 f1 bộ binhE3 f1 bộ binh
E3 f1 bộ binh
 
E2 f1 bộ binh
E2 f1 bộ binhE2 f1 bộ binh
E2 f1 bộ binh
 
E1 f1 bộ binh
E1 f1 bộ binhE1 f1 bộ binh
E1 f1 bộ binh
 
Nữ quái sân trườngtruonghocso.com
Nữ quái sân trườngtruonghocso.comNữ quái sân trườngtruonghocso.com
Nữ quái sân trườngtruonghocso.com
 
Những chàng trai xấu tính nguyễn nhật ánhtruonghocso.com
Những chàng trai xấu tính  nguyễn nhật ánhtruonghocso.comNhững chàng trai xấu tính  nguyễn nhật ánhtruonghocso.com
Những chàng trai xấu tính nguyễn nhật ánhtruonghocso.com
 
Những bài văn mẫu dành cho học sinh lớp 10truonghocso.com
Những bài văn mẫu dành cho học sinh lớp 10truonghocso.comNhững bài văn mẫu dành cho học sinh lớp 10truonghocso.com
Những bài văn mẫu dành cho học sinh lớp 10truonghocso.com
 

19 phương phap chứng minh bất đẳng thức

  • 1. 19 Phương pháp chứng minh Bất đẳng thức_ Nguyễn Đức Hàn www.VNMATH.com PHẦN 1 CÁC KIẾN THỨC CẦN LƯU Ý A ≥ B ⇔ A − B ≥ 0 1/Định nghĩa  A ≤ B ⇔ A − B ≤ 0 2/Tính chất + A>B ⇔ B < A + A>B và B >C ⇔ A > C + A>B ⇒ A+C >B + C + A>B và C > D ⇒ A+C > B + D + A>B và C > 0 ⇒ A.C > B.C + A>B và C < 0 ⇒ A.C < B.C + 0 < A < B và 0 < C <D ⇒ 0 < A.C < B.D + A > B > 0 ⇒ A n > B n ∀n + A > B ⇒ A n > B n với n lẻ + A > B ⇒ A n > B n với n chẵn + m > n > 0 và A > 1 ⇒ A m > A n + m > n > 0 và 0 <A < 1 ⇒ A m < A n 1 1 +A < B và A.B > 0 ⇒ > A B 3/Một số hằng bất đẳng thức + A 2 ≥ 0 với ∀ A ( dấu = xảy ra khi A = 0 ) + An ≥ 0 với ∀ A ( dấu = xảy ra khi A = 0 ) + A ≥ 0 với ∀A (dấu = xảy ra khi A = 0 ) + -A <A= A + A + B ≥ A + B ( dấu = xảy ra khi A.B > 0) + A − B ≤ A − B ( dấu = xảy ra khi A.B < 0) Sưu tầm và tuyển chọn 1
  • 2. 19 Phương pháp chứng minh Bất đẳng thức_ Nguyễn Đức Hàn www.VNMATH.com PHẦN II CÁC PHƯƠNG PHÁP CHỨNG MINH BẤT ĐẲNG THỨC Phương pháp 1 : Dùng định nghĩa Kiến thức : Để chứng minh A > B. Ta lập hiệu A –B > 0 Lưu ý dùng hằng bất đẳng thức M 2 ≥ 0 với∀ M Ví dụ 1 ∀ x, y, z chứng minh rằng : a) x 2 + y 2 + z 2 ≥ xy+ yz + zx b) x 2 + y 2 + z 2 ≥ 2xy – 2xz + 2yz c) x 2 + y 2 + z 2 +3 ≥ 2 (x + y + z) Giải: 1 a) Ta xét hiệu : x 2 + y 2 + z 2 - xy – yz – zx = .2 .( x 2 + y 2 + z 2 - xy – yz – zx) 2 = 1 2 [ ] ( x − y ) 2 + ( x −z ) 2 + ( y − z ) 2 ≥ 0 đúng với mọi x;y;z ∈ R Vì (x-y)2 ≥ 0 với∀x ; y Dấu bằng xảy ra khi x=y (x-z)2 ≥ 0 với∀x ; z Dấu bằng xảy ra khi x=z (y-z)2 ≥ 0 với∀ z; y Dấu bằng xảy ra khi z=y Vậy x 2 + y 2 + z 2 ≥ xy+ yz + zx. Dấu bằng xảy ra khi x = y =z b)Ta xét hiệu: x 2 + y 2 + z 2 - ( 2xy – 2xz +2yz ) = x 2 + y 2 + z 2 - 2xy +2xz –2yz = ( x – y + z) 2 ≥ 0 đúng với mọi x;y;z ∈ R Vậy x 2 + y 2 + z 2 ≥ 2xy – 2xz + 2yz đúng với mọi x;y;z ∈ R Dấu bằng xảy ra khi x+y=z c) Ta xét hiệu: x 2 + y 2 + z 2 +3 – 2( x+ y +z ) = x 2 - 2x + 1 + y 2 -2y +1 + z 2 -2z +1 = (x-1) 2 + (y-1) 2 +(z-1) 2 ≥ 0. Dấu(=)xảy ra khi x=y=z=1 Ví dụ 2: chứng minh rằng : 2 2 a2 + b2  a + b  a2 + b2 + c2  a + b + c  a) ≥  ; b) ≥  c) Hãy tổng quát bài toán 2  2  3  3  Giải: 2 a2 + b2  a + b  a) Ta xét hiệu −  2  2  2( a 2 + b 2 ) a 2 + 2ab + b 2 = ( 2a 2 + 2b 2 − a 2 − b 2 − 2ab ) = ( a − b ) ≥ 0 1 1 2 = − 4 4 4 4 2 a +b 2 2 a+b Vậy ≥  . Dấu bằng xảy ra khi a=b 2  2  b)Ta xét hiệu Sưu tầm và tuyển chọn 2
  • 3. 19 Phương pháp chứng minh Bất đẳng thức_ Nguyễn Đức Hàn www.VNMATH.com [ ] 2 a2 + b2 + c2  a + b + c  1  = ( a − b ) + ( b − c ) + ( c − a ) ≥ 0 .Vậy 2 2 2 − 3  3  9 2 a2 + b2 + c2  a + b + c  ≥  3  3  Dấu bằng xảy ra khi a = b =c c)Tổng quát 2 a12 + a 2 + .... + a n  a1 + a 2 + .... + a n  2 2 ≥  n  n  Tóm lại các bước để chứng minh A ≥ B theo định nghĩa Bước 1: Ta xét hiệu H = A - B Bước 2:Biến đổi H=(C+D) 2 hoặc H=(C+D) 2 +….+(E+F) 2 Bước 3:Kết luận A ≥ B Ví dụ 1: Chứng minh ∀m,n,p,q ta đều có : m 2 + n 2 + p 2 + q 2 +1≥ m(n+p+q+1) Giải:  m2   m2   m2   m2  ⇔  4 − mn + n 2  +    4 − mp + p 2  +    4 − mq + q 2  +    4 − m + 1 ≥ 0          2 2 2 2 m  m  m  m  ⇔  − n  +  − p  +  − q  +  − 1 ≥ 0 (luôn đúng) 2  2  2  2  m  2 −n =0  m m n = 2  − p=0  m 2 p =  m=2 Dấu bằng xảy ra khi  m ⇔ 2 ⇔  −q =0  m n = p = q = 1 2  q= m m =2 2 −1 = 0  2  Ví dụ 2: Chứng minh rằng với mọi a, b, c ta luôn có : a 4 + b 4 + c 4 ≥ abc(a + b + c) Giải: Ta có : a 4 + b 4 + c 4 ≥ abc(a + b + c) , ∀ , b, c > 0 a ⇔ a 4 + b 4 + c 4 − a 2 bc − b 2 ac − c 2 ab ≥ 0 ⇔ 2a 4 + 2b 4 + 2c 4 − 2a 2 bc − 2b 2 ac − 2c 2 ab ≥ 0 ( ⇔ a2 −b2 ) 2 ( + 2a 2 b 2 + b 2 − c 2 ) 2 ( + 2b 2 c 2 + c 2 − a 2 ) 2 + 2a 2 c 2 − 2a 2 bc − 2b 2 ac − 2c 2 ab ≥ 0 ( ⇔ a2 −b2 ) + (b 2 2 −c2 ) + (c 2 2 −a2 ) 2 + (a 2 b 2 + b 2 c 2 − 2b 2 ac ) + (b 2 c 2 + c 2 a 2 − 2c 2 ab) + ( a 2 b 2 + c 2 a 2 − 2a 2 ab) ≥ 0 ( ⇔ a2 −b2 ) + (b 2 2 −c2 ) + (c 2 2 −a2 ) + ( ab − bc ) 2 2 + ( bc − ac ) + ( ab − ac ) ≥ 0 2 2 Đúng với mọi a, b, c. Phương pháp 2 : Dùng phép biến đổi tương đương Sưu tầm và tuyển chọn 3
  • 4. 19 Phương pháp chứng minh Bất đẳng thức_ Nguyễn Đức Hàn www.VNMATH.com Kiến thức: Ta biến đổi bất đẳng thức cần chứng minh tương đương với bất đẳng thức đúng hoặc bất đẳng thức đã được chứng minh là đúng. Nếu A < B ⇔ C < D , với C < D là một bất đẳng thức hiển nhiên, hoặc đã biết là đúng thì có bất đẳng thức A < B . Chú ý các hằng đẳng thức sau: ( A + B ) 2 = A 2 + 2 AB + B 2 ( A + B + C ) 2 = A 2 + B 2 + C 2 + 2 AB + 2 AC + 2 BC ( A + B ) 3 = A 3 + 3 A 2 B + 3 AB 2 + B 3 Ví dụ 1: Cho a, b, c, d,e là các số thực chứng minh rằng b2 a) a 2 + ≥ ab 4 b) a 2 + b 2 + 1 ≥ ab + a + b c) a 2 + b 2 + c 2 + d 2 + e 2 ≥ a( b + c + d + e ) Giải: b2 ≥ ab ⇔ 4a 2 + b 2 ≥ 4ab ⇔ 4a 2 − 4a + b 2 ≥ 0 ⇔ ( 2a − b ) ≥ 0 2 a) a 2 + 4 b2 (BĐT này luôn đúng). Vậy a 2 + ≥ ab (dấu bằng xảy ra khi 2a=b) 4 b) a + b + 1 ≥ ab + a + b ⇔ 2(a + b 2 + 1 ) > 2(ab + a + b) 2 2 2 ⇔ a 2 − 2ab + b 2 + a 2 − 2a + 1 + b 2 − 2b + 1 ≥ 0 ⇔ (a − b) 2 + (a − 1) 2 + (b − 1) 2 ≥ 0 Bất đẳng thức cuối đúng. Vậy a 2 + b 2 + 1 ≥ ab + a + b . Dấu bằng xảy ra khi a=b=1 c) a 2 + b 2 + c 2 + d 2 + e 2 ≥ a( b + c + d + e ) ⇔ 4( a 2 + b 2 + c 2 + d 2 + e 2 ) ≥ 4a( b + c + d + e ) ⇔ ( a 2 − 4ab + 4b 2 ) + ( a 2 − 4ac + 4c 2 ) + ( a 2 − 4ad + 4d 2 ) + ( a 2 − 4ac + 4c 2 ) ≥ 0 ⇔ ( a − 2b ) 2 + ( a − 2c ) 2 + ( a − 2d ) 2 + ( a − 2c ) 2 ≥ 0 Bất đẳng thức đúng vậy ta có điều phải chứng minh Ví dụ 2: Chứng minh rằng: ( a10 + b10 )( a 2 + b 2 ) ≥ ( a 8 + b 8 )( a 4 + b 4 ) Giải: (a + b10 )( a 2 + b 2 ) ≥ ( a 8 + b 8 )( a 4 + b 4 ) ⇔ a 12 + a 10 b 2 + a 2 b10 + b12 ≥ a 12 + a 8 b 4 + a 4 b 8 + b12 10 ⇔ a 8 b 2 ( a 2 − b 2 ) + a 2 b 8 ( b 2 − a 2 ) ≥ 0 ⇔ a2b2(a2-b2)(a6-b6) ≥ 0 ⇔ a2b2(a2-b2)2(a4+ a2b2+b4) ≥ 0 Bất đẳng thứccuối đúng vậy ta có điều phải chứng minh x2 + y2 Ví dụ 3: cho x.y =1 và x 〉 y Chứng minh ≥2 2 x− y x2 + y2 Giải: ≥ 2 2 vì :x 〉 y nên x- y 〉 0 ⇒ x2+y2 ≥ 2 2 ( x-y) x− y ⇒ x +y2- 2 2 x+ 2 2 y ≥ 0 ⇔ x2+y2+2- 2 2 x+ 2 2 y -2 ≥ 0 2 ⇔ x2+y2+( 2 )2- 2 2 x+ 2 2 y -2xy ≥ 0 vì x.y=1 nên 2.x.y=2 ⇒ (x-y- 2 )2 ≥ 0 Điều này luôn luôn đúng . Vậy ta có điều phải chứng minh Ví dụ 4: Chứng minh rằng: a/ P(x,y)= 9 x 2 y 2 + y 2 − 6 xy − 2 y + 1 ≥ 0 ∀x, y ∈ R Sưu tầm và tuyển chọn 4
  • 5. 19 Phương pháp chứng minh Bất đẳng thức_ Nguyễn Đức Hàn www.VNMATH.com b/ a 2 + b 2 + c 2 ≤ a + b + c (gợi ý :bình phương 2 vế) c/ Cho ba số thực khác không x, y, z thỏa mãn:  x. y.z = 1 1 1 1  + + < x+ y+z x y z  Chứng minh rằng :có đúng một trong ba số x,y,z lớn hơn 1 Giải: Xét (x-1)(y-1)(z-1)=xyz+(xy+yz+zx)+x+y+z-1 1 1 1 1 1 1 1 1 1 =(xyz-1)+(x+y+z)-xyz( x + y + z )=x+y+z - ( + + ) > 0 (vì x + y + z < x+y+z x y z theo gt) ⇒ 2 trong 3 số x-1 , y-1 , z-1 âm hoặc cả ba sỗ-1 , y-1, z-1 là dương. Nếu trường hợp sau xảy ra thì x, y, z >1 ⇒ x.y.z>1 Mâu thuẫn gt x.y.z=1 bắt buộc phải xảy ra trường hợp trên tức là có đúng 1 trong ba số x ,y ,z là số lớn hơn 1 a b c Ví dụ 5: Chứng minh rằng : 1 < a + b + b + c + a + c < 2 Giải: 1 1 a a Ta có : a+b< a+b+c⇒ > ⇒ > (1) a+b a+b+c a+b a+b+c b b c c Tương tự ta có : b + c > a + b + c (2) , a + c > a + b + c (3) Cộng vế theo vế các bất đẳng thức (1), (2), (3), ta được : a b c + + > 1 (*) a+b b+c a+c a a+c Ta có : a < a + b ⇒ a + b < a + b + c (4) b a+b c c+b Tương tự : b + c < a + b + c (5) , < c+a a+b+c ( 6) Cộng vế theo vế các bất đẳng thức (4), (5), (6), ta được : a b c + + <2 (**) a+b b+c a+c a b c Từ (*) và (**) , ta được : 1 < a + b + b + c + a + c < 2 (đpcm) Phương pháp 3: Dùng bất đẳng thức phụ Kiến thức: a) x 2 + y 2 ≥ 2 xy b) x + y ≥ xy dấu( = ) khi x = y = 0 2 2 c) ( x + y ) 2 ≥ 4 xy a b d) + ≥ 2 b a Ví dụ 1 Cho a, b ,c là các số không âm chứng minh rằng (a+b)(b+c)(c+a) ≥ 8abc Giải: Dùng bất đẳng thức phụ: ( x + y ) 2 ≥ 4 xy Sưu tầm và tuyển chọn 5
  • 6. 19 Phương pháp chứng minh Bất đẳng thức_ Nguyễn Đức Hàn www.VNMATH.com Tacó ( a + b ) 2 ≥ 4ab ; ( b + c ) 2 ≥ 4bc ; ( c + a ) 2 ≥ 4ac ⇒ ( a + b ) 2 ( b + c ) 2 ( c + a ) 2 ≥ 64a 2 b 2 c 2 = ( 8abc ) 2 ⇒ (a+b)(b+c)(c+a) ≥ 8abc Dấu “=” xảy ra khi a = b = c Phương pháp 4: Bất đẳng thức Cô sy Kiến thức: a/ Với hai số không âm : a, b ≥ 0 , ta có: a + b ≥ 2 ab . Dấu “=” xảy ra khi a=b b/ Bất đẳng thức mở rộng cho n số không âm : a1 + a 2 + ... + a n ≥ n n a1 a 2 ..a n n  a + a 2 + ... + a n  ⇔ a1 a 2 ..a n ≤  1   n  Dấu “=” xảy ra khi a1 = a 2 = ... = a n Chú ý : ta dùng bất đẳng thức Côsi khi đề cho biến số không âm. 2x 4x 2x 3 Ví dụ 1 : Giải phương trình : + x + x = 4 +1 2 +1 2 + 4 x x 2  a = 2x Giải : Nếu đặt t =2x thì pt trở thành pt bậc 6 theo t nên ta đặt  , a, b > 0  b = 4 x a b 1 3 Khi đó phương trình có dạng : b + 1 + a + 1 + a + b = 2 Vế trái của phương trình:  a   b   1  = + 1 +  + 1 +  + 1 − 3  b +1   a +1   a + b   a + b +1  a + b +1  a + b +1 = + + −3  b +1   a +1   a + b   1 1 1  = ( a + b + c) + + −3  b +1 a +1 a + b  [ ( b + 1) + ( a + 1) + ( a + b ) ] 1 + 1 + 1  − 3    b +1 a +1 a + b  1 3 3 3 ≥ 3 ( a + 1)( b + 1)( a + b ) . −3 = 2 3 ( a + 1)( b + 1)( a + b ) 2 Vậy phương trình tương đương với : a + 1 = b + 1 = a + b ⇔ a = b = 1 ⇔ 2x = 4x = 1 ⇔ x = 0 . x y z Ví dụ 2 : Cho x, y , z > 0 và x + y + z = 1. Tìm GTLN của P = x + 1 + y + 1 + z + 1 1 1 1 Giải : P = 3- ( x + 1 + y + 1 + z + 1 ) = 3 – Q. Theo BDT Côsi , nếu a, b, c > 0 thì Sưu tầm và tuyển chọn 6
  • 7. 19 Phương pháp chứng minh Bất đẳng thức_ Nguyễn Đức Hàn www.VNMATH.com a + b + c ≥ 3 3 abc 1 1 1 1 + + ≥ 33 a b c abc 1 1 1 ⇒ ( a + b + c ) + +  ≥ 9 a b c 1 1 1 9 ⇒ + + ≥ a b c a +b+c 1 1 1 9 9 9 3 Suy ra Q = x + 1 + y + 1 + z + 1 ≥ 4 ⇒ -Q ≤ − 4 nên P = 3 – Q ≤ 3- 4 = 4 3 1 Vậy max P = 4 .khi x = y = z = 3 . 1 1 1 a+b+c Ví dụ 3: Cho a, b, c >0 . Chứng minh rằng: + 2 + 2 ≤ a + bc b + ac c + ab 2 2abc Giải: Áp dụng bất đẳng thức Côsi ta có : 2 1 1 1 1  a 2 + +bc ≥ 2a bc ⇒ ≤ ≤  +  a + +bc a bc 2  ab ac  2 Tương tự : 2 1 1 1 1  ≤ ≤  +  b + + ac b ac 2  bc ab  2 2 1 1 1 1 ≤ ≤  +  c + + ab c ab 2  ac bc  2 2 2 2 a+b+c ⇒ 2 + 2 + 2 ≤ a + bc b + + ac c + + ab 2abc Dấu “=” xảy ra khi a = b = c. a b c Ví dụ 4 : CMR trong tam giác ABC : b + c − a + c + a − b + a + b − c ≥ 3 (*) Giải : Theo bất đẳng thức Côsi : a b c abc + + ≥ 33 (1) b + c − a c + a −b a +b −c (b + c − a )(c + a − b)(a + b − c ) Cũng theo bất đẳng thức Côsi : 1 (b + c − a )(c + a − b) ≤ (b + c − a + c + a − b) = c (2) 2 Viết tiếp hai BDT tương tự (2) rồi nhân với nhau sẽ được (b + c − a )(c + a − b)(a + b − c) ≤ abc abc → ≥ 1 (3) (b + c − a )(c + a − b)(a + b − c ) Từ (1),(3) suy ra (*). Dấu “=” xảy ra khi a = b = c hay ABC là đều . Ví dụ 5: 0< a≤ b≤ c ( a + c) 2 Cho  . Chứng minh rằng: ( + by + cz )  + +  ≤ x y z   ( x + y + z) 2  0 < x, y , z a b c 4ac Giải: Đặt f ( x) = x 2 − (a + c) x + ac = 0 có 2 nghiệm a,c Mà: a ≤ b ≤ c ⇒ f (b) ≤ 0 ⇔ b 2 − (a + c)b + ac ≤ 0 Sưu tầm và tuyển chọn 7
  • 8. 19 Phương pháp chứng minh Bất đẳng thức_ Nguyễn Đức Hàn www.VNMATH.com ac y ⇔b+ ≤ a + c ⇔ yb + ac ≤ ( a + c ) y b b  x y z ⇒  xa + ac  + ( yb + ac ) + ( zc + ac ) ≤ ( a + c ) x + ( a + c ) y + ( a + c) z  a b c x y z ⇒ xa + yb + zc + ac + +  ≤ ( a + c )( x + y + z ) a b c Theo bất đẳng thức Cauchy ta có: ⇒2 ( xa + yb + zc ) ac x + y + z  ≤ ( a + c )( x + y + z )   a b c x y z ⇔ 4( xa + yb + zc ) ac + +  ≤ ( a + c ) ( x + y + z ) 2 2 a b c  x y z  ( a + c) 2 ⇔ ( xa + yb + zc ) ac + +  ≤ ( x + y + z ) 2 (đpcm) a b c 4ac Phương pháp 5 Bất đẳng thức Bunhiacopski Kiến thức: Cho 2n số thực ( n ≥ 2 ): a1 , a 2 ,...a n , b1 , b2 ,..., bn . Ta luôn có: (a1b1 + a 2 b2 + ... + a n bn ) ≤ (a12 + a 2 + ... + a n )(b12 + b2 + ... + bn ) 2 2 2 2 2 a a a Dấu “=” xảy ra khi ⇔ b = b = .... = b 1 2 n 1 2 n b1 b2 bn Hay a = a = .... = a (Quy ước : nếu mẫu = 0 thì tử = 0 ) 1 2 n Chứng minh:  a = a 2 + a 2 + ... + a 2  1 2 n Đặt   b = b1 + b2 + ... + bn 2 2 2  • Nếu a = 0 hay b = 0: Bất đẳng thức luôn đúng. • Nếu a,b > 0: ai b Đặt: α i = , β i = i ( i = 1,2,...n ) , Thế thì: α 12 + α 22 + ... + α n2 = β 12 + β 22 + ... + β n2 a b Mặt khác: α i β i ≤ 2 (α i + β i2 ) 1 2 1 2 1 α1 β1 + α 2 β 2 + ... + α n β n ≤ (α1 + α 2 + .... + α n ) + ( β12 + β 22 + ... + β n2 ) ≤ 1 2 2 Suy ra: 2 2 ⇒ a1b1 + a 2 b2 + ... + a n bn ≤ a.b Lại có: a1b1 + a 2 b2 +... + a n bn ≤ a1b1 + a 2 b2 +... + a n bn Suy ra: (a1b1 + a 2 b2 + ... + a n bn ) 2 ≤ (a12 + a 2 + ... + a n )(b12 + b2 + ... + bn ) 2 2 2 2  α i = β i ( ∀i = 1,2,..., n ) a a a Dấu”=” xảy ra ⇔  ⇔ 1 = 2 = .... = n α 1 β 1 ....α n β n cùng dáu b1 b2 bn Ví dụ 1 : 1 Chứng minh rằng: ∀x ∈R , ta có: sin 8 x + cos 8 x ≥ 8 Giải: Ta có: sin 2 x + cos 2 x = 1, ∀x ∈ R Theo bất đẳng thức Bunhiacopski, ta có: Sưu tầm và tuyển chọn 8
  • 9. 19 Phương pháp chứng minh Bất đẳng thức_ Nguyễn Đức Hàn www.VNMATH.com 1 = (sin 2 x.1 + cos 2 x.1) ≤ (sin 4 x + cos 4 x )(12 + 12 ) 1 ⇔ ≤ sin 4 x + cos 4 x 2 ⇒ ≤ (sin 4 x + cos 4 x ) 1 2 4 Sử dụng bất đẳng thức Bunhiacopski một lần nữa: ≤ (sin 4 x.1 + cos 4 x.1) 1 2 ⇔ 4 ⇔ ≤ (sin 8 x + cos 8 x )(12 + 12 ) 1 4 ⇔ (sin 4 x + cos 4 x ) ≥ 1 8 Ví dụ 2: Cho tam giác ABC có các góc A,B,C nhọn. Tìm GTLN của: P = 1 + tan A. tan B + 1 + tan B. tan C + 1 + tan C. tan A Giải: * Bất đẳng thức Bunhiacopski mở rộng Cho m bộ số, mỗi bộ số gồm n số không âm: ( a i , bi ,..., ci )(i =1,2,...., m) Thế thì: (a1 a 2 ...a m + b1b2 ...bm + ... + c1c 2 ...c m ) 2 ≤ (a1m + b1m + ... + c1m )(a 2 + b2 + ... + c 2 )(a m + bm + ... + c m ) m m m m m m Dấu”=” xảy ra ⇔ ∃ bô số (a,b,….,c) sao cho: với mỗi i = 1,2,…,m thì ∃ t i sao cho: a = t i ai , b = t i bi ,..., c = t i ci , Hay a1 : b1 : ... : c1 = a 2 : b2 : ... : c 2 = a n : bn : ...cn a12 + a 2 + ... + a n = 3 2 2 Ví dụ 1: Cho   n ∈ Z,n ≥ 2 a1 a 2 a Chứng minh rằng: + + .... + n < 2 2 3 n +1 Giải: 1 1 1 < = ∀ ∈N k * ta có: k2 1  1  1 k2 −  k −  k +  4  2  2 1 1 1 ⇒ < − k2 1 1 k− k+ 2 2   1 1 1  1 1   1 1   1 1  ⇒ 2 + 2 + ... + 2 <       3 − 5  +  5 − 7  + ... +  −      1 1 n+  2 3 n  2 2  2 2  n−   2 2 1 1 2 = − < 3 1 3 2 n+ 2 Do đó theo bất đẳng thức Bunhiacopski: a1 a 2 a 1 1 1 2 + + .... + n ≤ a12 + a 2 + ... + a n 2 2 + 2 + ... + 2 < 3 < 2 (đpcm) 2 3 n +1 2 2 3 n 3 Ví dụ 2: Cho 4 số a,b,c,d bất kỳ chứng minh rằng: ( a + c) 2 + (b + d ) 2 ≤ a 2 + b 2 + c 2 + d 2 Giải: Dùng bất đẳng thức Bunhiacopski: Tacó ac+bd ≤ a 2 + b 2 . c 2 + d 2 Sưu tầm và tuyển chọn 9
  • 10. 19 Phương pháp chứng minh Bất đẳng thức_ Nguyễn Đức Hàn www.VNMATH.com mà ( a + c ) 2 + ( b + d ) 2 = a 2 + b 2 + 2( ac + bd ) + c 2 + d 2 ≤ ( a 2 + b 2 ) + 2 a 2 + b 2 . c 2 + d 2 + c 2 + d 2 ⇒ (a + c) 2 + (b + d ) 2 ≤ a 2 + b 2 + c 2 + d 2 Ví dụ 3: Chứng minh rằng : a 2 + b 2 + c 2 ≥ ab + bc + ac Giải: Dùng bất đẳng thức Bunhiacopski Cách 1: Xét cặp số (1,1,1) và (a,b,c) ta có (12 + 12 + 12 )(a 2 + b 2 + c 2 ) ≥ (1.a + 1.b + 1.c ) 2 ⇒ 3 ( a 2 + b 2 + c 2 ) ≥ a 2 + b 2 + c 2 + 2( ab + bc + ac ) ⇒ a 2 + b 2 + c 2 ≥ ab + bc + ac Điều phải chứng minh Dấu bằng xảy ra khi a=b=c Phương pháp 6: Bất đẳng thức Trê- bư-sép Kiến thức:  a1 ≤ a2 ≤ ..... ≤ an a1 + a 2 + ... + a n b1 + b2 + .... + bn a1b1 + a 2 b2 + .... + a n bn a)Nếu  thì . ≤ .  b1 ≤ b2 ≤ ..... ≤ bn n n n a1 = a 2 = .... = a n Dấu ‘=’ xảy ra khi và chỉ khi b  1 = b2 = .... = bn  a1 ≤ a2 ≤ ..... ≤ an b)Nếu  thì  b1 ≥ b2 ≥ ..... ≥ bn a1 + a 2 + ... + a n b1 + b2 + .... + bn a1b1 + a 2 b2 + .... + a n bn . ≥ n n n a1 = a 2 = .... = a n Dấu ‘=’ xảy ra khi và chỉ khi b  1 = b2 = .... = bn Ví dụ 1: Cho ∆ABC có 3 góc nhọn nội tiếp đường tròn bán kính R = 1 và sin A. sin 2a + sin B. sin 2 B + sin C. sin 2C 2 S = . sin A + sin B + sin C 3 S là diện tích tan giác. chứng minh rằng ∆ABC là tam giác đều. π Giải: Không giảm tính tổng quát ta giả sư 0< A≤B≤C < . Suy ra: 2  sin A ≤ sin B ≤ sin C   sin 2a ≤ sin 2B ≤ sin 2C Áp dụng BĐT trebusep ta được: ( sin A + sin B + sin C )( sin 2 A + sin 2 B + sin 2C ) ≥ ≥ 3( sin A. sin 2 A + sin B. sin 2 B + sin C. sin 2C ) sin A. sin 2 A + sin B. sin 2 B + sin C. sin 2C 1 ⇔ ≤ (sin 2 A + sin 2 B + sin 2C ) sin A + sin B + sin C 3 sin A = sin B = sin C Dấu ‘=’ xảy ra ⇔ sin 2 A = sin 2 B = sin 2C ⇔ ∆ABC dêu  Mặt khác: Sưu tầm và tuyển chọn 10
  • 11. 19 Phương pháp chứng minh Bất đẳng thức_ Nguyễn Đức Hàn www.VNMATH.com sin 2 A + sin 2 B + sin 2C = 2 sin( A + B ). cos( A − B ) + sin 2C = 2 sin C [cos( A − B ) + cos C ] = 2 sin C [cos( A − B ) − cos( A + B )] = 2 sin C.2 sin A. sin B = 4 sin A sin B sin C = (2 R sin A)(2 R sin B ). sin C = a.b. sin C = 2 S (2) Thay (2) vào (1) ta có sin A. sin 2a + sin B. sin 2 B + sin C. sin 2C 2S ≤ . sin A + sin B + sin C 3 Dấu ‘=’ xảy ra ⇔ ∆ ABC đều. Ví dụ 2(HS tự giải): 1 1 1 a/ Cho a,b,c>0 và a+b+c=1 CMR: + + ≥9 a b c b/ Cho x,y,z>0 và x+y+z=1 CMR:x+2y+z ≥ 4(1 − x)(1 − y )(1 − z ) c/ Cho a>0 , b>0, c>0 a b c 3 CMR: + + ≥ b+c c+a a+b 2 1 d)Cho x ≥ 0 ,y ≥ 0 thỏa mãn 2 x − y = 1 ;CMR: x+y ≥ 5 a3 b3 c3 1 Ví dụ 3: Cho a>b>c>0 và a 2 + b 2 + c 2 = 1 . Chứng minh rằng + + ≥ b+c a+c a+b 2 Giải:   a2 ≥ b2 ≥ c2 Do a,b,c đối xứng ,giả sử a ≥ b ≥ c ⇒  a ≥ b ≥ c b + c a + c a + b  Áp dụng BĐT Trê- bư-sép ta có a b c a2 + b2 + c2  a b c  1 3 1 a2. + b2. + c2. ≥ . + + = . = b+c a+c a+b 3 b+c a+c a+b 3 2 2 a3 b3 c3 1 1 Vậy + + ≥ Dấu bằng xảy ra khi a=b=c= b+c a+c a+b 2 3 Ví dụ 4: Cho a,b,c,d>0 và abcd =1 .Chứng minh rằng : a + b + c + d 2 + a( b + c ) + b( c + d ) + d ( c + a ) ≥ 10 2 2 2 Giải: Ta có a 2 + b 2 ≥ 2ab c 2 + d 2 ≥ 2cd 1 1 1 Do abcd =1 nên cd = (dùng x + ≥ ) ab x 2 1 Ta có a 2 + b 2 + c 2 ≥ 2(ab + cd ) = 2(ab + ) ≥ 4 (1) ab Mặt khác: a( b + c ) + b( c + d ) + d ( c + a ) = (ab+cd)+(ac+bd)+(bc+ad)  1   1   1  =  ab +  +  ac +  +  bc +  ≥ 2 + 2 + 2  ab   ac   bc  Vậy a + b + c + d + a( b + c ) + b( c + d ) + d ( c + a ) ≥ 10 2 2 2 2 Phương pháp7 Bất đẳng thức Bernouli Kiến thức: Sưu tầm và tuyển chọn 11
  • 12. 19 Phương pháp chứng minh Bất đẳng thức_ Nguyễn Đức Hàn www.VNMATH.com a)Dạng nguyên thủy: Cho a ≥ -1, 1 ≤ n ∈ Z thì (1 + a ) n ≥ 1 + na . Dấu ‘=’ xảy ra khi  =0 a và chỉ khi n =1  b) Dạng mở rộng: - Cho a > -1, α ≥ 1 thì (1 + a ) α ≥ 1 + na . Dấu bằng xảy ra khi và chỉ khi a = 0.  =0 a - cho a ≥ −1,0 < α < 1 thì (1 + a ) α ≤ 1 + na . Dấu bằng xảy ra khi va chỉ khi  =1 . α  Ví dụ 1 : Chứng minh rằng a b + b a > 1, ∀a, b > 0 . Giải - Nếu a ≥ 1 hay b ≥ 1 thì BĐT luôn đúng - Nếu 0 < a,b < 1 Áp dụng BĐT Bernouli: b(1 − a ) a + b b b 1  1− a  = 1+   <1+  < a  a  a a a ⇒ ab > . a +b b Chứng minh tương tự: b a > . Suy ra a b + b a > 1 (đpcm). a+b Ví dụ 2: Cho a,b,c > 0.Chứng minh rằng 5 a5 + b5 + c5  a + b + c  3 ≥ 3  . (1)   Giải 5 5 5 (1) ⇔  3a  +  3b  +  3c  ≥ 3       a+b+c a+b+c a+b+c Áp dụng BĐT Bernouli: 5( b + c − 2a ) 5 5  3a   b + c − 2a    = 1 + a +b+c a +b+c   ≥1+ a +b+c (2)   Chứng minh tương tự ta đuợc: 5( c + a − 2b ) 5  3b    ≥1+ a +b +c  a +b +c (3)  5( a + b − 2c ) 5  3c    ≥1+ a +b +c  a +b +c (4)  Cộng (2) (3) (4) vế theo vế ta có 5 5 5  3a   3b   3c    +  +  ≥ 3 ⇒ (đpcm) a+b+c a+b+c a+b+c Chú ý: ta có bài toán tổng quát sau đây: “Cho a1 , a 2 ,...a n > 0; r ≥1. Chứng minh rằng r a1r + a 2 + .... + a n  a1 + a 2 + .... + a n  r r ≥   .  n  n  Dấu ‘=’ ⇔ a1 = a 2 = .... = a n .(chứng minh tương tự bài trên). Ví dụ 3: Cho 0 ≤ x, y, z ≤1 . Chứng minh rằng (2 x + 2 y + 2 z )( 2 − x + 2 − y + 2 − z ) ≤ 81 8 . Giải Đặt a = 2 x , b = 2 y , c = 2 z (1 ≤ a, b, c ≤ 2 ) . Sưu tầm và tuyển chọn 12
  • 13. 19 Phương pháp chứng minh Bất đẳng thức_ Nguyễn Đức Hàn www.VNMATH.com 1 ≤ a ≤ 2 ⇒ ( a − 1)( a − 2 ) ≤ 0 2 ⇒ a 2 − 3a + 2 ≤ 0 ⇒ a + ≤ 3 (1) a Chứng minh tương tự: 2 b+ ≤3 ( 2) b 2 c+ ≤3 (3) c Cộng (1) (2) (3) vế theo vế ta được  1 1 1  côsi  1 1 1 9 ≥ ( a + b + c ) + 2 + +  ≥ 2 ( a + b + c ) 2 + +  a b c a b c 81  1 1 1 ⇒ ≥ (a + b + c) + +  ⇒ (đpcm) 8 a b c Chú ý: Bài toán tổng quát dạng này “ Cho n số x1 , x 2 ,...., x n ∈[ a, b] , c > 1 Ta luôn có: ) ≤ [ n( c + cb ) ] 2 (c )( c a − x1 − x2 − xn x1 + c + .... + c x2 xn +c + .... + c 4c a + b Phương pháp 8: Sử dụng tính chất bắc cầu Kiến thức: A>B và B>C thì A>C Ví dụ 1: Cho a, b, c ,d >0 thỏa mãn a> c+d , b>c+d Chứng minh rằng ab >ad+bc Giải: a > c + d a − c > d > 0 Tacó  ⇒  ⇒ (a-c)(b-d) > cd b > c + d b − d > c > 0 ⇔ ab-ad-bc+cd >cd ⇔ ab> ad+bc (điều phải chứng minh) 5 1 1 1 1 Ví dụ 2: Cho a,b,c>0 thỏa mãn a + b + c = . Chứng minh 2 2 2 + + < 3 a b c abc Giải: Ta có :( a+b- c) = a +b +c +2( ab –ac – bc) 〉 0 2 2 2 2 1 2 2 2 ⇒ ac+bc-ab 〈 ( a +b +c ) 2 5 1 1 1 1 ⇒ ac+bc-ab ≤ 〈 1 Chia hai vế cho abc > 0 ta có + − 〈 6 a b c abc Ví dụ 3: Cho 0 < a,b,c,d <1 Chứng minh rằng (1-a).(1-b) ( 1-c).(1-d) > 1-a-b-c-d Giải: Ta có (1-a).(1-b) = 1-a-b+ab Do a>0 , b>0 nên ab>0 ⇒ (1-a).(1-b) > 1-a-b (1) Do c <1 nên 1- c >0 ta có ⇒ (1-a).(1-b) ( 1-c) > 1-a-b-c ⇒ (1-a).(1-b) ( 1-c).(1-d) > (1-a-b-c) (1-d) =1-a-b-c-d+ad+bd+cd ⇒ (1-a).(1-b) ( 1-c).(1-d) > 1-a-b-c-d (Điều phải chứng minh) Ví dụ 4: Cho 0 <a,b,c <1 . Chứng minh rằng: 2a 3 + 2b 3 + 2c 3 < 3 + a 2 b + b 2 c + c 2 a Sưu tầm và tuyển chọn 13
  • 14. 19 Phương pháp chứng minh Bất đẳng thức_ Nguyễn Đức Hàn www.VNMATH.com Giải: Do a < 1 ⇒ a 2 < 1 và Ta có (1 − a 2 ).(1 − b ) < 0 ⇒ 1-b- a 2 + a 2 b > 0 ⇒ 1+ a 2 b 2 > a 2 + b mà 0< a,b <1 ⇒ a 2 > a 3 , b 2 > b 3 Từ (1) và (2) ⇒ 1+ a 2 b 2 > a 3 + b 3 . Vậy a 3 + b 3 < 1+ a 2 b 2 Tương tự b 3 + c 3 ≤ 1 + b 2 c c 3 + a3 ≤ 1 + c2a Cộng các bất đẳng thức ta có : 2a 3 + 2b 3 + 2c 3 ≤ 3 + a 2 b + b 2 c + c 2 a Ví dụ 5 Chứng minh rằng : Nếu a 2 + b 2 = c 2 + d 2 = 1998 thì ac+bd =1998 Giải: Ta có (ac + bd) 2 + (ad – bc ) 2 = a 2 c 2 + b 2 d 2 + 2abcd + a 2 d 2 + b 2 c 2 - 2abcd = = a2(c2+d2)+b2(c2+d2) =(c2+d2).( a2+ b2) = 19982 rõ ràng (ac+bd)2 ≤ ( ac + bd ) 2 + ( ad − bc ) 2 = 1998 2 ⇒ ac + bd ≤ 1998 Ví dụ 6 (HS tự giải) : a/ Cho các số thực : a1; a2;a3 ….;a2003 thỏa mãn : a1+ a2+a3 + ….+a2003 =1 1 c hứng minh rằng : a 1 + a 2 + a32 + .... + a 2003 ≥ 2 2 2 2003 b/ Cho a;b;c ≥ 0 thỏa mãn :a+b+c=1 1 1 1 Chứng minh rằng: ( − 1).( − 1).( − 1) ≥ 8 a b c Phương pháp 9: Dùng tính chất của tỷ số Kiến thức 1) Cho a, b ,c là các số dương thì a a a+c a – Nếu > 1 thì > b b b+c a a a+c b – Nếu < 1 thì < b b b+c 2) Nếu b,d >0 thì từ a c a a+c c < ⇒ < < b d b b+d d ` Ví dụ 1: Cho a,b,c,d > 0 .Chứng minh rằng a b c d 1< + + + <2 a+b+c b+c+d c+d +a d +a+b Giải: Theo tính chất của tỉ lệ thức ta có a a a+d <1⇒ < (1) a+b+c a+b+c a+b+c+d a a Mặt khác : > (2) a+b+c a+b+c+d Từ (1) và (2) ta có a a a+d < < (3) a+b+c+d a+b+c a+b+c+d Sưu tầm và tuyển chọn 14
  • 15. 19 Phương pháp chứng minh Bất đẳng thức_ Nguyễn Đức Hàn www.VNMATH.com Tương tự ta có b b b+a < < (4) a+b+c+d b+c+d a+b+c+d c c b+c < < (5) a+b+c+d c+d +a a+b+c+d d d d +c < < (6) a+b+c+d d +a+b a+b+c+d cộng vế với vế của (3); (4); (5); (6) ta có a b c d 1< + + + < 2 điều phải chứng minh a+b+c b+c+d c+d +a d +a+b a c a ab + cd c Ví dụ 2 :Cho: < và b,d > 0 .Chứng minh rằng < 2 < b d b b +d2 d a c ab cd ab ab + cd cd c Giải: Từ < ⇒ 2 < 2 ⇒ 2 < 2 < = b d b d b b +d2 d2 d a ab + cd c Vậy < < điều phải chứng minh b b2 + d 2 d Ví dụ 3 : Cho a;b;c;dlà các số nguyên dương thỏa mãn : a+b = c+d =1000 a b tìm giá trị lớn nhất của + c d a b a b a a+b b Giải: Không mất tính tổng quát ta giả sử : ≤ Từ : ≤ ⇒ ≤ ≤ c d c d c c+d d a ≤ 1 vì a+b = c+d c b a b a/ Nếu :b ≤ 998 thì ≤ 998 ⇒ + ≤ 999 d c d a b 1 999 b/Nếu: b=998 thì a=1 ⇒ + = + Đạt giá trị lớn nhất khi d= 1; c=999 c d c d a b 1 Vậy giá trị lớn nhất của + =999+ khi a=d=1; c=b=999 c d 999 Phương pháp 10: Phương pháp làm trội Kiến thức: Dùng các tính bất đẳng thức để đưa một vế của bất đẳng thức về dạng tính được tổng hữu hạn hoặc tích hữu hạn. (*) Phương pháp chung để tính tổng hữu hạn : S = u1 + u2 + .... + un Ta cố gắng biến đổi số hạng tổng quát u k về hiệu của hai số hạng liên tiếp nhau: u k = ak − ak +1 Khi đó :S = ( a1 − a2 ) + ( a2 − a3 ) + .... + ( an − an+1 ) = a1 − an+1 (*) Phương pháp chung về tính tích hữu hạn: P = u1u2 ....un k a Biến đổi các số hạng u k về thương của hai số hạng liên tiếp nhau: u k = a k +1 a a a a Khi đó P = a . a ..... a = a 1 2 n 1 2 3 n +1 n +1 Sưu tầm và tuyển chọn 15
  • 16. 19 Phương pháp chứng minh Bất đẳng thức_ Nguyễn Đức Hàn www.VNMATH.com Ví dụ 1: Với mọi số tự nhiên n >1 chứng minh rằng 1 1 1 1 3 < + + .... + < 2 n +1 n + 2 n+n 4 1 1 1 Giải: Ta có > = với k = 1,2,3,…,n-1 n + k n + n 2n 1 1 1 1 1 n 1 Do đó: + + ... + > + ... + = = n +1 n + 2 2n 2n 2 n 2n 2 Ví dụ 2: Chứng minh rằng: 1 2 1+ + 1 3 + .... + 1 n ( > 2 n +1 −1 ) Với n là số nguyên Giải: Ta có 1 = k 2 k 2 > 2 k + k +1 = 2 k +1 − k ( ) Khi cho k chạy từ 1 đến n ta có 1 > 2 ( 2 − 1) 1 2 >2 3− 2( ) ……………… 1 n ( > 2 n +1 − n ) Cộng từng vế các bất đẳng thức trên ta có 1 + 1 2 + 1 3 + .... + 1 n ( ) > 2 n +1 −1 n 1 Ví dụ 3: Chứng minh rằng ∑k k =1 2 <2 ∀n ∈ Z 1 1 1 1 Giải: Ta có k 2 < k ( k − 1) = k − 1 − k Cho k chạy từ 2 đến n ta có 1 1 2 < 1− 2 2 1 1 1 < − 32 2 3 ................. 1 1 1 < − n 2 n −1 n 1 1 1 ⇒ 2 + 2 + .... + 2 < 1 2 3 n n 1 Vậy ∑k k =1 2 <2 Phương pháp 11: Dùng bất đẳng thức trong tam giác Kiến thức: Nếu a;b;clà số đo ba cạnh của tam giác thì : a;b;c> 0 Sưu tầm và tuyển chọn 16
  • 17. 19 Phương pháp chứng minh Bất đẳng thức_ Nguyễn Đức Hàn www.VNMATH.com Và |b-c| < a < b+c ; |a-c| < b < a+c ; |a-b| < c < b+a Ví dụ 1: Cho a;b;c là số đo ba cạnh của tam giác chứng minh rằng 1/ a2+b2+c2< 2(ab+bc+ac) 2/ abc>(a+b-c).(b+c-a).(c+a-b) Giải 1/Vì a,b,c là số đo 3 cạnh của một tam giác nên ta có 0 < a < b + c a 2 < a (b + c)   2 0 < b < a + c ⇒ b < b( a + c ) 0 < c < a + b  c 2 < c ( a + b)   Cộng từng vế các bất đẳng thức trên ta có: a2+b2+c2< 2(ab+bc+ac) 2/ Ta có a > b-c  ⇒ a 2 > a 2 − (b − c) 2 > 0 b > a-c  ⇒ b 2 > b 2 − (c − a ) 2 > 0 c > a-b  ⇒ c 2 > c 2 − ( a − b) 2 > 0 Nhân vế các bất đẳng thức ta được [ ][ ][ ⇒ a 2b 2 c 2 > a 2 − ( b − c ) b 2 − ( c − a ) c 2 − ( a − b ) 2 2 2 ] ⇒ a 2b 2 c 2 > ( a + b − c ) ( b + c − a ) ( c + a − b ) 2 2 2 ⇒ abc > ( a + b − c ).( b + c − a ).( c + a − b ) Ví dụ2 (HS tự giải) 1/ Cho a,b,c là chiều dài ba cạnh của tam giác Chứng minh rằng ab + bc + ca < a 2 + b 2 + c 2 < 2(ab + bc + ca) 2/Cho a,b,c là chiều dài ba cạnh của tam giác có chu vi bằng 2 Chứng minh rằng a 2 + b 2 + c 2 + 2abc < 2 Phương pháp 12: Sử dụng hình học và tọa độ Ví dụ 1: Chứng minh rằng : c(a −c) + c(b −c) ≤ ab , ∀a ≥ b ≥ 0 và b ≥ c Giải Trong mặt phẳng Oxy, chọn u =( c, b −c ) ; v =( a −c , c ) Thì u = b , v = a ; u.v = c ( a −c ) + c (b −c ) Hơn nữa: u.v =u . v . cos(u , v ) ≤u . v ⇒ c ( a − ) + c (b − ) ≤ ab ⇒ c c (ĐPCM) Ví dụ 2: n n Cho 2n số: xi ; y i , i =1,2,..., n thỏa mãn: ∑ xi + ∑ yi = 1. Chứng minh rằng: i =1 i =1 n 2 ∑ i =1 xi2 + yi2 ≥ 2 Giải: Vẽ hình y MN MK H Sưu tầm và tuyển chọn 17
  • 18. 19 Phương pháp chứng minh Bất đẳng thức_ Nguyễn Đức Hàn www.VNMATH.com M 1 x O x+y=1 Trong mặt phẳng tọa độ, xét: M 1 ( x1 , y1 ) : M 2 ( x1 + x 2 , y1 + y 2 ) ;…; M n ( x1 +  + x n , y1 +  + y n ) Giả thiết suy ra M n ∈ đường thẳng x + y = 1. Lúc đó: OM 1 = x12 + y12 , M 1M 2 = x2 + y 2 2 2 , M 2M 3 = x3 + y 3 2 2 ,…, M n −1 M n = xn + y n 2 2 2 Và OM 1 + M 1 M 2 + M 2 M 3 +  + M n −1 M n ≥ OM n ≥ OH = 2 n 2 ⇒ ∑ i =1 xi2 + yi2 ≥ 2 ⇒ (ĐPCM) Phương pháp 13: Đổi biến số a b c 3 Ví dụ1: Cho a,b,c > 0 Chứng minh rằng + + ≥ (1) b+c c+a a+b 2 y+z−x z+x− y x+ y−z Giải: Đặt x=b+c ; y=c+a ;z= a+b ta có a= ; b= ;c= 2 2 2 y+z−x z+x− y x+ y−z 3 ta có (1) ⇔ + + ≥ 2x 2y 2z 2 y z x z x y y x z x z y ⇔ + −1+ + −1+ + −1 ≥ 3 ⇔ ( + ) + ( + ) + ( + ) ≥ 6 x x y y z z x y x z y z y x z x z y Bất đẳng thức cuối cùng đúng vì ( x + y ≥ 2; + ≥ 2; + ≥ 2 nên ta có điều y z x z phải chứng minh Ví dụ2: Cho a,b,c > 0 và a+b+c <1. Chứng minh rằng 1 1 1 + 2 + 2 ≥9 (1) a + 2bc b + 2ac c + 2ab 2 Giải: Đặt x = a 2 + 2bc ; y = b 2 + 2ac ; z = c 2 + 2ab . Ta có x + y + z = ( a + b + c ) 2 < 1 1 1 1 (1) ⇔ x + y + z ≥ 9 Với x+y+z < 1 và x ,y,z > 0 1 1 1 1 Theo bất đẳng thức Côsi ta có: x + y + z ≥ 3. 3 xyz , và: x + y + z ≥ 3. 3 xyz ( x + y + z ). 1 + 1 + 1  ≥ 9 . Mà x+y+z < 1. Vậy 1 1 1 ⇒  x y z  + + ≥9 (đpcm)   x y z 1 Ví dụ3: Cho x ≥ 0 , y ≥ 0 thỏa mãn 2 x − y = 1 CMR x + y ≥ 5 Gợi ý: Đặt x = u , y =v ⇒ 2u-v =1 và S = x+y = u 2 + v 2 ⇒ v = 2u-1 Sưu tầm và tuyển chọn 18
  • 19. 19 Phương pháp chứng minh Bất đẳng thức_ Nguyễn Đức Hàn www.VNMATH.com thay vào tính S min Bài tập tự giải 25a 16b c 1) Cho a > 0 , b > 0 , c > 0 CMR: + + >8 b+c c+a a+b 2)Tổng quát m, n, p, q, a, b >0 CMR ma + nb + pc b+c c+a a+b 2 ≥ 1 ( ) m + n + p − ( m + n + p) 2 Phương pháp 14: Dùng tam thức bậc hai Kiến thứ: Cho f(x) = ax2 + bx + c Định lí 1: a> 0 f(x) > 0, ∀ x ⇔  ∆ < 0 a> 0 f (x) ≥ 0, ∀ x ⇔  ∆ ≤ 0 a< 0 f (x) < 0, ∀ x ⇔  ∆ < 0 a< 0 f (x) ≤ 0, ∀ x ⇔  ∆ ≤ 0 Định lí 2: Phương trình f(x) = 0 có 2 nghiệm x1 < α < x2 ⇔ a. f ( α ) < 0 Phương trình f(x) = 0 có 2 nghiệm :   a. f ( α ) > 0  x1 < x2 < α ⇔  ∆ > 0 S  <α 2 Phương trình f(x) = 0 có 2 nghiệm : Sưu tầm và tuyển chọn 19
  • 20. 19 Phương pháp chứng minh Bất đẳng thức_ Nguyễn Đức Hàn www.VNMATH.com   a. f ( α ) > 0  α < x1 < x2 ⇔  ∆ > 0 S  >α 2 α < x1 < β < x 2 Phương trình f(x) = 0 có 2 nghiệm  x < α < x < β ⇔ f (α ). f ( β ) < 0.  1 2 Ví dụ 1:Chứng minh rằng f ( x, y ) = x 2 + 5 y 2 − 4 xy + 2 x − 6 y + 3 > 0 (1) Giải: Ta có (1) ⇔ x 2 − 2 x( 2 y − 1) + 5 y 2 − 6 y + 3 > 0 ∆′ = ( 2 y − 1) − 5 y 2 + 6 y − 3 2 = 4 y2 − 4 y +1− 5y2 + 6 y − 3 = −( y − 1) − 1 < 0 2 Vậy f ( x, y ) > 0 với mọi x, y Ví dụ2: Chứng minh rằng: f ( x, y ) = x 2 y 4 + 2( x 2 + 2). y 2 + 4 xy + x 2 > 4 xy 3 Giải: Bất đẳng thức cần chứng minh tương đương với x 2 y 4 + 2( x 2 + 2 ). y 2 + 4 xy + x 2 − 4 xy 3 > 0 ⇔ ( y 2 + 1) 2 .x 2 + 4 y (1 − y ) 2 x + 4 y 2 > 0 Ta có ∆′ = 4 y 2 (1 − y 2 ) − 4 y 2 ( y 2 + 1) = −16 y 2 < 0 2 2 Vì a = ( y 2 + 1) > 0 vậy f ( x, y ) > 0 2 (đpcm) Phương pháp 15: Dùng quy nạp toán học Kiến thức: Để chứng minh bất đẳng thức đúng với n > n0 ta thực hiện các bước sau : 1 – Kiểm tra bất đẳng thức đúng với n = n0 2 - Giả sử BĐT đúng với n =k (thay n =k vào BĐT cần chứng minh được gọi là giả thiết quy nạp ) 3- Ta chứng minh bất đẳng thức đúng với n = k +1 (thay n = k+1vào BĐT cần chứng minh rồi biến đổi để dùng giả thiết quy nạp) 4 – kết luận BĐT đúng với mọi n > n0 1 1 1 1 Ví dụ1: Chứng minh rằng : 2 + 2 + .... + 2 < 2 − ∀n ∈ N ; n > 1 (1) 1 2 n n 1 1 Giải: Với n =2 ta có 1 + < 2 − (đúng). Vậy BĐT (1) đúng với n =2 4 2 Giả sử BĐT (1) đúng với n =k ta phải chứng minh BĐT (1) đúng với n = k+1 1 1 1 1 1 Thật vậy khi n =k+1 thì (1) ⇔ 12 + 22 + .... + k 2 + (k + 1) 2 < 2 − k + 1 Theo giả thiết quy nạp Sưu tầm và tuyển chọn 20
  • 21. 19 Phương pháp chứng minh Bất đẳng thức_ Nguyễn Đức Hàn www.VNMATH.com 1 1 1 1 1 1 1 ⇔ + 2 + .... + 2 + < 2− + < 2− 2 1 2 k (k + 1) 2 k ( k + 1) 2 k +1 1 1 1 1 1 ⇔ + .... + < + < 12 (k + 1) 2 k + 1 ( k + 1) 2 k k +1+1 1 ⇔ < ⇔ k (k + 2) < (k + 1) 2 ⇔ k2+2k<k2+2k+1 Điều này đúng .Vậy bất (k + 1) 2 k đẳng thức (1)được chứng minh n  a+b an + bn Ví dụ2: Cho n ∈ N và a+b> 0. Chứng minh rằng   ≤ (1)  2  2 Giải: Ta thấy BĐT (1) đúng với n=1 Giả sử BĐT (1) đúng với n=k ta phải chứng minh BĐT đúng với n=k+1 Thật vậy với n = k+1 ta có k +1  a+b a k +1 + b k +1 (1) ⇔   ≤  2  2 k a+b a+b a k +1 + b k +1 ⇔   . ≤ (2)  2  2 2 a k + b k a + b a k +1 + ab k + a k b + b k +1 a k +1 + b k +1 ⇔ Vế trái (2) ≤ . = ≤ 2 2 4 2 k +1 k +1 k +1 k +1 ≥ 0 ⇔ ( a k − b k ).( a − b ) ≥ 0 a +b a + ab + a b + b k k ⇔ − (3) 2 4 Ta chứng minh (3) (+) Giả sử a ≥ b và giả thiết cho a ≥ -b ⇔ a ≥ b k ⇔ a k ≥ b ≥ bk ⇒ (a k − b k ).( a − b ) ≥ 0 (+) Giả sử a < b và theo giả thiết - a<b ⇔ a < b k ⇔ a k < b k ⇔ ( a k − b k ).( a − b ) ≥ 0 k Vậy BĐT (3)luôn đúng ta có (đpcm) Ví dụ 3: Cho a ≥ −1 ,1 ≤ n ∈Ν. Chứng minh rằng : (1 + a) n ≥ 1 + n.a Giải n=1: bất đẳng thức luôn đúng n=k ( k ∈ Ν ): giả sử bất đẳng thức đúng, tức là: (1 + a ) k ≥ 1 + k .a n= k+1 . Ta cần chứng minh: (1 + a ) k +1 ≥ 1 + (k +1).a Ta có: (1 + a) k +1 = (1 + a).(1 + a) k ≥ (1 + a).(1 + k .a ) ≥ 1 + (k +1)a + k .a 2 ≥ 1 + (k +1)a ⇒ Bất đẳng thức đúng với n= k+1 V ậy theo nguyên lý quy nạp: (1 + a) n ≥ 1 + n.a , ∀n ∈Ν 1 Ví dụ 4: Cho 1 ≤ n ∈ Ν a1 , a 2 ,  , a n ≥ 0 thoả mãn a1 + a 2 +  + a n ≤ . Chứng minh 2 1 rằng: (1 − a1 )(1 − a 2 )  (1 − a n ) ≥ 2 1 1 Giải n=1: a1 ≤ ⇒ 1 − a1 ≥ ⇒ Bài toán đúng 2 2 1 n=k ( k ∈ Ν ): giả sử bất đẳng thức đúng, tức là: (1 − a1 )(1 − a 2 )  (1 − a k ) ≥ 2 Sưu tầm và tuyển chọn 21
  • 22. 19 Phương pháp chứng minh Bất đẳng thức_ Nguyễn Đức Hàn www.VNMATH.com 1 n= k+1 . Ta cần chứng minh: (1 − a1 )(1 − a 2 )  (1 − a k +1 ) ≥ 2 Ta có: (1 − a1 )(1 − a 2 )  (1 − a k +1 ) = (1 − a1 )(1 − a 2 )  (1 − a k −1 )[1 − (a k + a k +1 ) + a k a k +1 ] 1 1 ≥ (1 − a1 )(1 − a 2 )  (1 − a k −1 )[1 − ( a k + a k +1 )] ≥ (Vì a1 + a 2 +  + a k −1 + (a k + a k +1 ) ≤ 2 ) 2 ⇒ Bất đẳng thức đúng với n= k+1 1 Vậy theo nguyên lý quy nạp: (1 − a1 )(1 − a 2 )  (1 − a n ) ≥ 2 Ví dụ 5: Cho 1 ≤ n ∈ Ν , a i , bi ∈R, i =1,2,..., n . Chứng minh rằng: (a1b1 + a 2 b2 +  + a n bn ) ≤ (a + a +  + a )(b + b +  + bn2 ) 2 2 1 2 2 2 n 1 2 2 2 Giải n=1: Bất đẳng thức luôn đúng n=k ( k ∈ Ν ):giả sử bất đẳng thức đúng, tức là: (a1b1 + a 2 b2 +  + a k bk ) 2 ≤ (a12 + a 2 +  + a k )(b12 + b22 +  + bk2 ) 2 2 n= k+1 . Ta cần chứng minh: (a1b1 + a 2 b2 +  + a k +1bk +1 ) 2 ≤ (a12 + a 2 +  + a k +1 )(b12 + b2 +  + bk2+1 ) (1) 2 2 2 Thật vậy: VP(1) = (a12 + a 22 +  + a k2 )(b12 + b22 +  + bk2 ) + (a12 +  + a k2 ).b 2 + + a 2 (b12 + b2 +  + bk2 ) + a k +1 .bk2+1 ≥ ( a1b1 + a 2 b2 +  + a k bk ) + 2a1b1 a k +1bk +1 + 2a 2 b2 a k +1bk +1 + 2 2 +  + 2a k bk a k +1bk +1 + a k +1bk +1 2 2 ≥ (a1b1 + a 2 b2 +  + a k bk ) 2 + 2 ( a1b1 + a 2 b2 +  + a k bk ) a k + bk + + a k +1 .bk +1 1 1 2 2 ≥ (a1b1 + a 2 b2 +  + a k +1bk +1 ) 2 Vậy (1) được chứng minh Ví dụ 6: Cho 1 ≤ n ∈ Ν , ai , bi ∈R, i =1,2,..., n . Chứng minh rằng: a1 + a 2 +  + a n 2 a12 + a 2 +  + a n 2 2 ( ) ≤ n n Giải: n=1: Bất đẳng thức luôn đúng a1 + a 2 +  + a k 2 a12 + a 2 +  + a k2 2 n=k ( k ∈ Ν ):giả sử bất đẳng thức đúng, tức là: ( ) ≤ k k a + a +  + a k +1 2 a1 + a 2 +  + a k +1 2 2 2 n= k+1 . Ta cần chứng minh: ( 1 2 ) ≤ (1) k +1 k +1 a + a +  + a k +1 Đặt: a = 2 3 k 1 VP (1) = (a12 + k 2 a 2 + 2ka1 a ) k +1 1  2 a 2 + a3 +  + a k2+1 2 a 2 + a3 +  + a k2+1  a12 + a 2 +  + a k2+1 2 2 ≥  a1 + k 2 2 + k .a12 + k 2 = (k + 1) 2  k k  k +1 Vậy (1) đựơc chứng minh Ví dụ 7: Chứng minh rằng: n n > ( n +1) n −1 , ∀n ∈ Ζ, n ≥ 2  n n = 4 Giải: n=2 ⇒  ⇒ n n > (n + 1) n −1  (n + 1) n− 1 = 3 n=k ≥ 2 : giả sử bất đẳng thức đúng, tức là: k k > (k + 1) k −1 n= k+1:Ta c ó: k k (k + 1) k +1 ≥ (k + 1) k −1 (k + 1) k +1 = (k + 1) 2 k −2 (k + 1) 2 = [(k + 1) 2 ] k −1 (k + 1) 2 Sưu tầm và tuyển chọn 22
  • 23. 19 Phương pháp chứng minh Bất đẳng thức_ Nguyễn Đức Hàn www.VNMATH.com > (k 2 + 2k ) k −1 ( k 2 + 2k ) (vì (k + 1) 2 = k 2 + 2k + 1 > k 2 + 2k ) ≥ k k ( k + 2) k ⇒ (k + 1) k +1 > (k + 2) k ⇒ Bất đẳng thức đúng với n= k+1 Vậy n n > (n +1) n−1 , ∀n ∈ Ζ, n ≥ 2 Ví dụ 8: Chứng minh rằng: sin nx ≤n sin x , ∀ ∈Ν , ∀ ∈R ∗ n x Giải: n=1: Bất đẳng thức luôn đúng n=k :giả sử bất đẳng thức đúng, tức là: sin kx ≤k sin x n= k+1 . Ta cần chứng minh: sin(k + ) x ≤(k + ) sin x 1 1  a + b ≤ a + b , ∀ a, b ∈ R  Ta có:   sin x , cos x ≤ 1, ∀ x ∈ R  Nên: sin( k +1) x = sin kx cos x +cos kx sin x ≤ sin kx . cos x + cos kx . sin x ≤ sin kx . + sin x ≤k sin x . +. sin x =( k + ) sin x . 1 ⇒ Bất đẳng thức đúng với n= k+1. Vậy: sin nx ≤n sin x , ∀ ∈ ∗, ∀ ∈ n Ν x R Phương pháp 16: Chứng minh phản chứng Kiến thức: 1) Giả sử phải chứng minh bất đẳng thức nào đó đúng , ta hãy giả sử bất đẳng thức đó sai và kết hợp với các giả thiết để suy ra điều vô lý , điều vô lý có thể là điều trái với giả thiết , có thể là điều trái ngược nhau .Từ đó suy ra bất đẳng thức cần chứng minh là đúng 2) Giả sử ta phải chứng minh luận đề “p ⇒ q” Muốn chứng minh p ⇒ q (với p : giả thiết đúng, q : kết luận đúng) phép chứng minh được thực hiên như sau: Giả sử không có q ( hoặc q sai) suy ra điều vô lý hoặc p sai. Vậy phải có q (hay q đúng) Như vậy để phủ định luận đề ta ghép tất cả giả thiết của luận đề với phủ định kết luận của nó . Ta thường dùng 5 hình thức chứng minh phản chứng sau : A - Dùng mệnh đề phản đảo : “P ⇒ Q” B – Phủ định rôi suy trái giả thiết C – Phủ định rồi suy trái với điều đúng D – Phủ định rồi suy ra 2 điều trái ngược nhau E – Phủ định rồi suy ra kết luận : Ví dụ 1: Cho ba số a,b,c thỏa mãn a +b+c > 0 , ab+bc+ac > 0 , abc > 0 Chứng minh rằng a > 0 , b > 0 , c > 0 Giải: Giả sử a ≤ 0 thì từ abc > 0 ⇒ a ≠ 0 do đó a < 0. Mà abc > 0 và a < 0 ⇒ cb < 0 Từ ab+bc+ca > 0 ⇒ a(b+c) > -bc > 0 Vì a < 0 mà a(b +c) > 0 ⇒ b + c < 0 a < 0 và b +c < 0 ⇒ a + b +c < 0 trái giả thiết a+b+c > 0 Sưu tầm và tuyển chọn 23
  • 24. 19 Phương pháp chứng minh Bất đẳng thức_ Nguyễn Đức Hàn www.VNMATH.com Vậy a > 0 tương tự ta có b > 0 , c > 0 Ví dụ 2:Cho 4 số a , b , c ,d thỏa mãn điều kiện ac ≥ 2.(b+d) .Chứng minh rằng có ít nhất một trong các bất đẳng thức sau là sai: a 2 < 4b , c 2 < 4d Giải: Giả sử 2 bất đẳng thức : a 2 < 4b , c 2 < 4d đều đúng khi đó cộng các vế ta được a 2 + c 2 < 4(b + d ) (1) Theo giả thiết ta có 4(b+d) ≤ 2ac (2) Từ (1) và (2) ⇒ a 2 + c 2 < 2ac hay ( a − c ) 2 < 0 (vô lý) Vậy trong 2 bất đẳng thức a 2 < 4b và c 2 < 4d có ít nhất một các bất đẳng thức sai Ví dụ 3:Cho x,y,z > 0 và xyz = 1. Chứng minh rằng 1 1 1 Nếu x+y+z > x + y + z thì có một trong ba số này lớn hơn 1 Giải :Ta có (x-1).(y-1).(z-1) =xyz – xy- yz + x + y+ z –1 1 1 1 1 1 1 =x + y + z – ( x + y + z ) vì xyz = theo giả thiết x+y +z > x + y + z nên (x-1).(y-1).(z-1) > 0 Trong ba số x-1 , y-1 , z-1 chỉ có một số dương Thật vậy nếu cả ba số dương thì x,y,z > 1 ⇒ xyz > 1 (trái giả thiết) Còn nếu 2 trong 3 số đó dương thì (x-1).(y-1).(z-1) < 0 (vô lý) Vậy có một và chỉ một trong ba số x , y,z lớn hơn 1 Ví dụ 4: Cho a, b, c >0 và a.b.c=1. Chứng minh rằng: a + b + c ≥ 3 (Bất đẳng thức Cauchy 3 số) Giải: Giả sử ngược l ại: a + b + c < 3 ⇒ (a + b + c)ab < 3ab ⇔ a 2 b + b 2 a + cab < 3ab ⇔ a 2 b + (a 2 − 3a )b + 1 < 0 Xét : f (b) = a 2 b + (a 2 − 3a)b + 1 Có ∆ = (a 2 − 3a) 2 − 4a = a 4 − 6a 3 + 9a 2 − 4a = a(a 3 − 6a 2 + 9a − 4) = = a(a −1) 2 (a − 4) ≤ 0  a , b, c > 0 (Vì  ⇒ 0 < a < 3 ) ⇒ f (b) ≥ 0 ⇒ vô lý. Vậy: a + b + c ≥ 3 a+ b+ c< 3 Ví dụ 5: Chứng minh rằng không tồn tại các số a, b, c đồng thời thỏa mãn (1),(2),(3): a < b −c (1) b < c −a (2) c < a −b (3) Giải: Giả sử tồn tại các số a, b, c đồng thời thỏa mãn (1),(2),(3), lúc đó: a < b −c ⇒ (b − c ) 2 > a 2 ⇒ −( a + b − c )(a − b + c) > 0 (1’) b < c −a ⇒ (c − a ) > b 2 2 ⇒ −(−a + b + c)(a + b − c ) > 0 (2’) c < a −b ⇒ ( a − b) 2 > c 2 ⇒ −(a + b − c)(−a + b + c) > 0 (3’) Nhân (1’), (2’) và (3’) vế với vế ta được: ⇒ −[(a + b − c)( a − b + c)(−a + b + c)] 2 > 0 ⇒ Vô lý. Vậy bài toán được chứng minh Phương pháp 17 : Sử dụng biến đổi lượng giác Sưu tầm và tuyển chọn 24
  • 25. 19 Phương pháp chứng minh Bất đẳng thức_ Nguyễn Đức Hàn www.VNMATH.com − π π  1. Nếu x ≤R thì đặt x = Rcos α , α ∈ [ 0, π ] ; hoặc x = Rsin α , α ∈ ,  2 2  R  π 2. Nếu x ≥R thì đặt x = α ∈[0, c ) ∪ π,3  cos α  2  x = a + R cosα 3.Nếu ( x − a ) 2 + ( y − b ) 2 = R 2 , ( > 0) thì đặt  , (α = 2π )  y = b + R sinα  x = α + aR cosα  , (α = 2π ) 2 2  x −α   y −β  4. Nếu   +  =R 2 a, b > 0 thì đặt  y = β + bR sinα  a   b  5. Nếu trong bài toán xuất hiện biểu thức : ( ax ) 2 + b 2 , ( a, b > 0) b  π π Thì đặt: x= tgα , α ∈  − ,  a  2 2 Ví dụ 1: Cmr : a 1 −b 2 +b 1 −a 2 + 3 ab − ( (1 −b )(1 −a ) ) ≤2, ∀ , b ∈[−1,1] 2 2 a Giải : a ≤1, b ≤1  a = cosα Đặt : (α , β ∈ [ 0, π ] )  b = cos β Khi đó : a 1 − b 2 + b 1 − a 2 + 3 ab − ( (1 − b )(1 − a ) ) 2 2 = cos α. sin β + cos β. sin α + 3 ( cos α. cos β − sin α. sin β ) = sin(α + β ) + 3. cos(α + β ) π = 2 cos(α + β − ) ∈[ − 2,2] ⇒ ( dpcm) 6 Ví dụ 2 : Cho a , b ≥1 .Chứng minh rằng : a b −1 + b a1 ≤ ab Giải :  1  a = cos 2 α    π  Đặt :  1  α , β ∈  0,     b =   2  cos 2 β  1 1 ⇒ a b −1 + b a −1 = tg 2 β + tg 2α cos α 2 cos 2 β tgβ tgα (tgβ . cos 2 β + tgα . cos 2 α ) = + = cos 2 α cos 2 β cos 2 β . cos 2 α 1 (sin 2 β + sin 2α ) sin(α + β ) cos in(α − β ) = = 2 cos 2 β . cos 2 α cos 2 β . cos 2 α 1 ≤ = ab cos β . cos 2 α 2 Sưu tầm và tuyển chọn 25
  • 26. 19 Phương pháp chứng minh Bất đẳng thức_ Nguyễn Đức Hàn www.VNMATH.com a 2 − (a − 4b) 2 Ví dụ 3: Cho ab ≠ 0 .Chứng minh rằng : − 2 2 − 2 ≤ ≤2 2−2 2 2 a + 4b Giải : a 2 − (a − 4b) 2 tg 2α − (tgα − 2) 2 ⇒ = a 2 + 4b 2 1 + tg 2α = 4(tgα − 1). cos 2 α  π π  Đặt: a = 2btgα , α ∈  − 2 , 22  = 2 sin 2α − 2(1 + cos 2α )   = 2(sin 2α − cos 2α ) − 2 π [ = 2 2 sin( 2α − ) − 2 ∈ − 2 2 − 2,2 2 − 2 2 ] Phương pháp 18: Sử dụng khai triển nhị thức Newton. Kiến thức: Công thức nhị thức Newton n ( a + b ) n ∑C nk a n −k b k , ∀n ∈ N * , ∀a, b ∈ R . k =0 n! Trong đó hệ số C n k = (0 ≤ ≤ ) k n . ( n − )! k! k Một số tính chất đặt biệt của khai triển nhị thức Newton: + Trong khai triển (a + b)n có n + 1 số hạng. + Số mũ của a giảm dần từ n đến 0, trong khi đó số mũ của b tăng từ 0 đến n. Trong mỗi số hạng của khai trtiển nhị thức Newton có tổng số mũ của a và b bằng n. +Các hệ số cách đều hai đầu thì bằng nhau C n = C n −k . k n + Số hạng thứ k + 1 là C nk a n −k .b k (0 ≤ k ≤ n) Ví dụ 1: Chứng minh rằng (1 + a ) n ≥ 1 + na, ∀a ≥ 0, ∀n ∈ N * (bất đẳng thức bernoulli) Giải n Ta có: (1 + a ) = ∑ C n a ≥ C n + C n a = 1 + na (đpcm) n k k 0 1 k =0 Ví dụ 2: Chứng minh rằng: n an +bn a +b a) 2 ≥  2  , ∀a, b ≥ 0, ∀n ∈ N *     n an +bn + cn  a +b + c  b) ≥  , ∀a, b, c ≥ 0, ∀n ∈ N * 3  3  Giải Theo công thức khai triển nhị thức Newton ta có: Sưu tầm và tuyển chọn 26